User login
Vax campaign averted nearly 140,000 U.S. deaths through early May: Study
New York had 11.7 fewer COVID-19 deaths per 10,000 adults, and Hawaii had 1.1 fewer deaths per 10,000 than would have occurred without vaccinations, the study shows. The rest of the states fell somewhere in between, with the average state experiencing five fewer COVID-19 deaths per 10,000 adults.
At a national level, this means that instead of the 550,000 COVID-19 deaths that occurred by early May, there would have been 709,000 deaths in the absence of a vaccination campaign, according to the study.
Researchers from RAND and Indiana University created models to estimate the number of COVID-19 deaths that would have happened without vaccinations. The difference between the actual number of deaths and those estimates provides a measure of the number of COVID-19 deaths averted by the vaccination campaign.
Information about vaccine doses administered in each state came from the Bloomberg COVID-19 Vaccine Tracker, and data on COVID-19 deaths for each state came from The New York Times’ Coronavirus (COVID-19) Data in the United States database.
The study spanned the period from Dec. 21, 2020 to May 9, 2021. The U.S. Food and Drug Administration issued its first emergency use authorization (EUA) for a COVID-19 vaccine to Pfizer/BioNTech on December 11, followed by an EUA for the Moderna vaccine on December 18 and one for Johnson & Johnson’s vaccine on Feb. 27, 2021.
Varied by state
There were wide variations in the speed and extent of the vaccination campaigns in various states, the researchers found. For example, West Virginia was the first state to reach 10 doses per 100 adults, reaching that goal on Jan. 16, 2021, and Idaho was the last state to hit that mark, on Feb. 4, 2021. Alaska was the first to reach 20 doses per 100 adults, on January 29, and Alabama was the last to do it, on February 21.
On May 6, California was the first state to administer 120 doses per 100 adults, but many states have still not reached that milestone.
The median number of days between the milestones of 10 and 20 doses per 100 adults was 19 days, and the median number of days between 20 and 40 doses per 100 adults was 24 days.
Hard to establish causality
The researchers emphasized that “establishment of causality is challenging” in comparing individual states’ vaccination levels with their COVID-19 mortality rates.
Aside from the study being observational, they pointed out, the analysis “relied on variation in the administration of COVID-19 vaccines across states … Vaccine administration patterns may be associated with declining mortality because of vaccine prevention of deaths and severe complications as state-level vaccine campaigns allocated initial doses to the highest-risk populations with the aim of immediately reducing COVID-19 deaths.”
Nevertheless, the authors note, “clinical trial evidence has shown that COVID-19 vaccines have high efficacy. Our study provides support for policies that further expand vaccine administration, which will enable larger populations to benefit.”
Study confirms vaccine benefit
Aaron Glatt, MD, chair of medicine at Mount Sinai South Nassau in Oceanside, New York, and a spokesman for the Infectious Disease Society of America, said in an interview that the study is important because it confirms the benefit of COVID-19 vaccination.
Regardless of whether the study’s results are statistically valid, he said, “I don’t think anyone can argue the benefit isn’t there. It’s a question of how important the benefit is.”
Dr. Glatt is not surprised that there are variations across states in the number of COVID-19 deaths averted through vaccination. “Clearly, in states where there was a lot of disease, a significant amount of vaccination is going to impact that tremendously.”
The authors note that their paper has some limitations. For one thing, they couldn’t determine what share of the estimated reduction in COVID-19 deaths was a result of the proportion of the population that was vaccinated or had antibodies and what share was a result of lower population-level risk for COVID-19 transmission.
Vaccination versus natural immunity
In addition, the researchers weren’t able to identify the roles of vaccination, natural immunity, and changes in mobility in the numbers of COVID-19 deaths.
Dr. Glatt says that’s understandable, since this was a retrospective study, and the researchers didn’t know how many people had been infected with COVID-19 at some point. Moreover, he adds, scientists don’t know how strong natural immunity from prior infection is, how long it endures, or how robust it is against new variants.
“It’s clear to me that there’s a benefit in preventing the second episode of COVID in people who had a first episode of COVID,” he said. “What we don’t know is how much that benefit is and how long it will last.”
The researchers also didn’t know how many people had gotten both doses of the Pfizer or the Moderna vaccine and how many of them had received only one. This is an important piece of information, Dr. Glatt said, but the lack of it doesn’t impair the study’s overall finding.
“Every vaccine potentially prevents death,” he stressed. “The more we vaccinate, the more deaths we’ll prevent. We’re starting to see increased vaccinations again. There were a million of them yesterday. So people are recognizing that COVID hasn’t gone away, and we need to vaccinate more people. The benefit from the vaccination hasn’t decreased. The more we vaccinate, the more the benefit will be.”
A version of this article first appeared on Medscape.com.
New York had 11.7 fewer COVID-19 deaths per 10,000 adults, and Hawaii had 1.1 fewer deaths per 10,000 than would have occurred without vaccinations, the study shows. The rest of the states fell somewhere in between, with the average state experiencing five fewer COVID-19 deaths per 10,000 adults.
At a national level, this means that instead of the 550,000 COVID-19 deaths that occurred by early May, there would have been 709,000 deaths in the absence of a vaccination campaign, according to the study.
Researchers from RAND and Indiana University created models to estimate the number of COVID-19 deaths that would have happened without vaccinations. The difference between the actual number of deaths and those estimates provides a measure of the number of COVID-19 deaths averted by the vaccination campaign.
Information about vaccine doses administered in each state came from the Bloomberg COVID-19 Vaccine Tracker, and data on COVID-19 deaths for each state came from The New York Times’ Coronavirus (COVID-19) Data in the United States database.
The study spanned the period from Dec. 21, 2020 to May 9, 2021. The U.S. Food and Drug Administration issued its first emergency use authorization (EUA) for a COVID-19 vaccine to Pfizer/BioNTech on December 11, followed by an EUA for the Moderna vaccine on December 18 and one for Johnson & Johnson’s vaccine on Feb. 27, 2021.
Varied by state
There were wide variations in the speed and extent of the vaccination campaigns in various states, the researchers found. For example, West Virginia was the first state to reach 10 doses per 100 adults, reaching that goal on Jan. 16, 2021, and Idaho was the last state to hit that mark, on Feb. 4, 2021. Alaska was the first to reach 20 doses per 100 adults, on January 29, and Alabama was the last to do it, on February 21.
On May 6, California was the first state to administer 120 doses per 100 adults, but many states have still not reached that milestone.
The median number of days between the milestones of 10 and 20 doses per 100 adults was 19 days, and the median number of days between 20 and 40 doses per 100 adults was 24 days.
Hard to establish causality
The researchers emphasized that “establishment of causality is challenging” in comparing individual states’ vaccination levels with their COVID-19 mortality rates.
Aside from the study being observational, they pointed out, the analysis “relied on variation in the administration of COVID-19 vaccines across states … Vaccine administration patterns may be associated with declining mortality because of vaccine prevention of deaths and severe complications as state-level vaccine campaigns allocated initial doses to the highest-risk populations with the aim of immediately reducing COVID-19 deaths.”
Nevertheless, the authors note, “clinical trial evidence has shown that COVID-19 vaccines have high efficacy. Our study provides support for policies that further expand vaccine administration, which will enable larger populations to benefit.”
Study confirms vaccine benefit
Aaron Glatt, MD, chair of medicine at Mount Sinai South Nassau in Oceanside, New York, and a spokesman for the Infectious Disease Society of America, said in an interview that the study is important because it confirms the benefit of COVID-19 vaccination.
Regardless of whether the study’s results are statistically valid, he said, “I don’t think anyone can argue the benefit isn’t there. It’s a question of how important the benefit is.”
Dr. Glatt is not surprised that there are variations across states in the number of COVID-19 deaths averted through vaccination. “Clearly, in states where there was a lot of disease, a significant amount of vaccination is going to impact that tremendously.”
The authors note that their paper has some limitations. For one thing, they couldn’t determine what share of the estimated reduction in COVID-19 deaths was a result of the proportion of the population that was vaccinated or had antibodies and what share was a result of lower population-level risk for COVID-19 transmission.
Vaccination versus natural immunity
In addition, the researchers weren’t able to identify the roles of vaccination, natural immunity, and changes in mobility in the numbers of COVID-19 deaths.
Dr. Glatt says that’s understandable, since this was a retrospective study, and the researchers didn’t know how many people had been infected with COVID-19 at some point. Moreover, he adds, scientists don’t know how strong natural immunity from prior infection is, how long it endures, or how robust it is against new variants.
“It’s clear to me that there’s a benefit in preventing the second episode of COVID in people who had a first episode of COVID,” he said. “What we don’t know is how much that benefit is and how long it will last.”
The researchers also didn’t know how many people had gotten both doses of the Pfizer or the Moderna vaccine and how many of them had received only one. This is an important piece of information, Dr. Glatt said, but the lack of it doesn’t impair the study’s overall finding.
“Every vaccine potentially prevents death,” he stressed. “The more we vaccinate, the more deaths we’ll prevent. We’re starting to see increased vaccinations again. There were a million of them yesterday. So people are recognizing that COVID hasn’t gone away, and we need to vaccinate more people. The benefit from the vaccination hasn’t decreased. The more we vaccinate, the more the benefit will be.”
A version of this article first appeared on Medscape.com.
New York had 11.7 fewer COVID-19 deaths per 10,000 adults, and Hawaii had 1.1 fewer deaths per 10,000 than would have occurred without vaccinations, the study shows. The rest of the states fell somewhere in between, with the average state experiencing five fewer COVID-19 deaths per 10,000 adults.
At a national level, this means that instead of the 550,000 COVID-19 deaths that occurred by early May, there would have been 709,000 deaths in the absence of a vaccination campaign, according to the study.
Researchers from RAND and Indiana University created models to estimate the number of COVID-19 deaths that would have happened without vaccinations. The difference between the actual number of deaths and those estimates provides a measure of the number of COVID-19 deaths averted by the vaccination campaign.
Information about vaccine doses administered in each state came from the Bloomberg COVID-19 Vaccine Tracker, and data on COVID-19 deaths for each state came from The New York Times’ Coronavirus (COVID-19) Data in the United States database.
The study spanned the period from Dec. 21, 2020 to May 9, 2021. The U.S. Food and Drug Administration issued its first emergency use authorization (EUA) for a COVID-19 vaccine to Pfizer/BioNTech on December 11, followed by an EUA for the Moderna vaccine on December 18 and one for Johnson & Johnson’s vaccine on Feb. 27, 2021.
Varied by state
There were wide variations in the speed and extent of the vaccination campaigns in various states, the researchers found. For example, West Virginia was the first state to reach 10 doses per 100 adults, reaching that goal on Jan. 16, 2021, and Idaho was the last state to hit that mark, on Feb. 4, 2021. Alaska was the first to reach 20 doses per 100 adults, on January 29, and Alabama was the last to do it, on February 21.
On May 6, California was the first state to administer 120 doses per 100 adults, but many states have still not reached that milestone.
The median number of days between the milestones of 10 and 20 doses per 100 adults was 19 days, and the median number of days between 20 and 40 doses per 100 adults was 24 days.
Hard to establish causality
The researchers emphasized that “establishment of causality is challenging” in comparing individual states’ vaccination levels with their COVID-19 mortality rates.
Aside from the study being observational, they pointed out, the analysis “relied on variation in the administration of COVID-19 vaccines across states … Vaccine administration patterns may be associated with declining mortality because of vaccine prevention of deaths and severe complications as state-level vaccine campaigns allocated initial doses to the highest-risk populations with the aim of immediately reducing COVID-19 deaths.”
Nevertheless, the authors note, “clinical trial evidence has shown that COVID-19 vaccines have high efficacy. Our study provides support for policies that further expand vaccine administration, which will enable larger populations to benefit.”
Study confirms vaccine benefit
Aaron Glatt, MD, chair of medicine at Mount Sinai South Nassau in Oceanside, New York, and a spokesman for the Infectious Disease Society of America, said in an interview that the study is important because it confirms the benefit of COVID-19 vaccination.
Regardless of whether the study’s results are statistically valid, he said, “I don’t think anyone can argue the benefit isn’t there. It’s a question of how important the benefit is.”
Dr. Glatt is not surprised that there are variations across states in the number of COVID-19 deaths averted through vaccination. “Clearly, in states where there was a lot of disease, a significant amount of vaccination is going to impact that tremendously.”
The authors note that their paper has some limitations. For one thing, they couldn’t determine what share of the estimated reduction in COVID-19 deaths was a result of the proportion of the population that was vaccinated or had antibodies and what share was a result of lower population-level risk for COVID-19 transmission.
Vaccination versus natural immunity
In addition, the researchers weren’t able to identify the roles of vaccination, natural immunity, and changes in mobility in the numbers of COVID-19 deaths.
Dr. Glatt says that’s understandable, since this was a retrospective study, and the researchers didn’t know how many people had been infected with COVID-19 at some point. Moreover, he adds, scientists don’t know how strong natural immunity from prior infection is, how long it endures, or how robust it is against new variants.
“It’s clear to me that there’s a benefit in preventing the second episode of COVID in people who had a first episode of COVID,” he said. “What we don’t know is how much that benefit is and how long it will last.”
The researchers also didn’t know how many people had gotten both doses of the Pfizer or the Moderna vaccine and how many of them had received only one. This is an important piece of information, Dr. Glatt said, but the lack of it doesn’t impair the study’s overall finding.
“Every vaccine potentially prevents death,” he stressed. “The more we vaccinate, the more deaths we’ll prevent. We’re starting to see increased vaccinations again. There were a million of them yesterday. So people are recognizing that COVID hasn’t gone away, and we need to vaccinate more people. The benefit from the vaccination hasn’t decreased. The more we vaccinate, the more the benefit will be.”
A version of this article first appeared on Medscape.com.
Some patients with more severe PH in COPD may respond to treatment
Patients with pulmonary hypertension (PH) as a complication of chronic obstructive pulmonary disease (COPD) have worse functional impairment and higher mortality, compared with patients who have idiopathic pulmonary arterial hypertension (IPAH).
Despite these factors, some patients with more severe PH in COPD may respond to treatment and show clinical improvement after treatment, according to recent research published in the journal CHEST®.
Carmine Dario Vizza, MD, of the pulmonary hypertension unit, department of cardiovascular and respiratory diseases at Sapienza University of Rome, and colleagues evaluated patients in the Comparative, Prospective Registry of Newly Initiated Therapies for Pulmonary Hypertension (COMPERA) database, enrolled up to August 2020, identifying 68 patients with moderate PH and COPD and 307 patients with severe PH and COPD. The researchers compared the PH and COPD groups with 307 patients who had idiopathic pulmonary arterial hypertension.
Overall, mostly older men made up the group of patients with moderate (50%; mean, 68.5 years) and severe PH in COPD (61%; mean 68.4 years), compared with those who had IPAH (37%; mean 61.7 years. Oral monotherapy for patients with PH and COPD was the main treatment, consisting of phosphodiesterase-5 inhibitors, while most patients with IPAH received endothelin receptor antagonists.
On functional tests, patients in the PH and COPD group tended to perform poorer on the 6-minute walking distance (6MWD) and World Health Organization functional class (WHO FC) than patients with IPAH. Specifically, among 42.7% of patients in both group for whom follow-up data were available, there was a similar frequency of improvement for 6MWD of 30 meters or more from baseline for both PH and COPD and IPAH groups (46.9% vs. 52.6%; P = .294), but there were significant differences between 6MWD between patients with moderate and severe PH and COPD (51.6% vs. 31.6%; P = .04). There was a nonsignificant improvement in WHO FC improvement of one or more classes for 65.6% of patients with PH and COPD and 58.3% of patients with IPAH with follow-up data available, with 28.5% of patients with PH in COPD improving compared with 35.8% of patients with IPAH (P = .078) and nonsignificant differences between moderate and severe PH and COPD (19.0% vs. 30.4%; P = .188).
Comparing outcomes
Follow-up data were available for 84% of patients with IPAH and 94% of patients with PH and COPD. Dr. Dario Vizza and colleagues found 45.7% of patients in the PH and COPD group and 24.9% of patients in the IPAH group died during follow-up, while 1.1% in the PH and COPD group and 1.5% of patients in the IPAH group underwent lung transplantations. For patients with moderate PH and COPD, 31.3% died and none underwent lung transplantation, while 49.0% of patients in the severe PH and COPD group died and 1.4% underwent lung transplantations.
Patients in the moderate PH and COPD group were more likely to discontinue treatment (10.9%), compared with patients with IPAH (6.6%) and patients with severe PH and COPD (5.2%). The most common reasons for discontinuations were tolerability and efficacy failure; the IPAH group had 63% of patients discontinue because of tolerability and 7% for efficacy failure, 47% of patients in the severe PH and COPD group discontinued because of tolerability and efficacy, and 29% discontinued treatment for tolerability and 57% for efficacy failure in the moderate and COPD group.
The researchers said male sex, low 6MWD, and high pulmonary vascular resistance at baseline were predictive of poorer outcomes for PH and COPD, but patients with more severe PH and COPD had better outcomes if they improved by 30 meters or more in 6MWD, or improved in WHO FC after receiving medical therapy. For patients with IPAH response to therapy was better among patients who were younger, had higher WHO FC, had high diffusing capacity of the lung for carbon monoxide, had high mean pulmonary artery pressure, and had low PCO2.
“Our data suggest that PH-targeted drug therapy in patients with COPD and severe PH may improve exercise tolerance and WHO FC in a subgroup of patients and that patients with COPD and PH who respond to therapy may have a better prognosis than patients who do not show clinical improvement. These findings need to be explored further in prospective, randomized controlled clinical studies,” the authors concluded.
More research needed
In a related editorial, James R. Klinger, MD, a pulmonologist with Brown University, Providence, R.I., and the director of the Rhode Island Hospital Pulmonary Hypertension Center in East Providence, said there is a “keen interest” in treating PH in COPD despite a lack of consistency on whether treatment is effective in this patient population. About 80% of PH centers in the United States treat PH in COPD when they treat conditions like lung disease with PAH medication, he pointed out. However, he questioned whether current medications designed for PAH could improve pulmonary hemodynamics for PH in COPD.
“Reasons that the pathobiologic condition of PH-COPD may differ from PAH include the likely exposure of the pulmonary circulation to greater degrees of hypoxia and hypercapnia and the greater loss of alveolar capillaries associated with emphysema,” he said.
The study by Dr. Dario Vizza and colleagues is an attempt to evaluate treatment response for patients with PH and COPD “in a way that allows comparison with patients who have been treated with similar drugs for PAH,” Dr. Klinger said. He noted the study’s retrospective nature, lack of control group, and lack of information on lung disease severity could limit the findings.
“These limitations preclude recommendations for the routine treatment of patients with PH-COPD, but the findings suggest that, despite greater morbidity at baseline, patients with PH-COPD may be nearly as likely to benefit from PAH medications as patients with IPAH,” he said.
“What is needed now is well-designed randomized controlled studies to determine whether improved outcomes can be achieved in this population and which patients are most likely to benefit,” he concluded. “Simply put: How bad does PH need to be in patients with COPD before treatment is helpful, and how severe does COPD need to be before PH treatment is futile?”
The authors reported personal and institutional relationships in the form of grants, consultancies, advisory board memberships, speakers bureau appointments, honoraria, patents, grant and research funding, lectures, travel compensation, and steering committee positions for a variety of pharmaceutical companies, agencies, societies, journals, medical publishing companies, and other organizations. Dr. Klinger reported his institution receives grant support from Acceleron and United Therapeutics in the area of PH, and he has been an unpaid consultant for Bayer.
Patients with pulmonary hypertension (PH) as a complication of chronic obstructive pulmonary disease (COPD) have worse functional impairment and higher mortality, compared with patients who have idiopathic pulmonary arterial hypertension (IPAH).
Despite these factors, some patients with more severe PH in COPD may respond to treatment and show clinical improvement after treatment, according to recent research published in the journal CHEST®.
Carmine Dario Vizza, MD, of the pulmonary hypertension unit, department of cardiovascular and respiratory diseases at Sapienza University of Rome, and colleagues evaluated patients in the Comparative, Prospective Registry of Newly Initiated Therapies for Pulmonary Hypertension (COMPERA) database, enrolled up to August 2020, identifying 68 patients with moderate PH and COPD and 307 patients with severe PH and COPD. The researchers compared the PH and COPD groups with 307 patients who had idiopathic pulmonary arterial hypertension.
Overall, mostly older men made up the group of patients with moderate (50%; mean, 68.5 years) and severe PH in COPD (61%; mean 68.4 years), compared with those who had IPAH (37%; mean 61.7 years. Oral monotherapy for patients with PH and COPD was the main treatment, consisting of phosphodiesterase-5 inhibitors, while most patients with IPAH received endothelin receptor antagonists.
On functional tests, patients in the PH and COPD group tended to perform poorer on the 6-minute walking distance (6MWD) and World Health Organization functional class (WHO FC) than patients with IPAH. Specifically, among 42.7% of patients in both group for whom follow-up data were available, there was a similar frequency of improvement for 6MWD of 30 meters or more from baseline for both PH and COPD and IPAH groups (46.9% vs. 52.6%; P = .294), but there were significant differences between 6MWD between patients with moderate and severe PH and COPD (51.6% vs. 31.6%; P = .04). There was a nonsignificant improvement in WHO FC improvement of one or more classes for 65.6% of patients with PH and COPD and 58.3% of patients with IPAH with follow-up data available, with 28.5% of patients with PH in COPD improving compared with 35.8% of patients with IPAH (P = .078) and nonsignificant differences between moderate and severe PH and COPD (19.0% vs. 30.4%; P = .188).
Comparing outcomes
Follow-up data were available for 84% of patients with IPAH and 94% of patients with PH and COPD. Dr. Dario Vizza and colleagues found 45.7% of patients in the PH and COPD group and 24.9% of patients in the IPAH group died during follow-up, while 1.1% in the PH and COPD group and 1.5% of patients in the IPAH group underwent lung transplantations. For patients with moderate PH and COPD, 31.3% died and none underwent lung transplantation, while 49.0% of patients in the severe PH and COPD group died and 1.4% underwent lung transplantations.
Patients in the moderate PH and COPD group were more likely to discontinue treatment (10.9%), compared with patients with IPAH (6.6%) and patients with severe PH and COPD (5.2%). The most common reasons for discontinuations were tolerability and efficacy failure; the IPAH group had 63% of patients discontinue because of tolerability and 7% for efficacy failure, 47% of patients in the severe PH and COPD group discontinued because of tolerability and efficacy, and 29% discontinued treatment for tolerability and 57% for efficacy failure in the moderate and COPD group.
The researchers said male sex, low 6MWD, and high pulmonary vascular resistance at baseline were predictive of poorer outcomes for PH and COPD, but patients with more severe PH and COPD had better outcomes if they improved by 30 meters or more in 6MWD, or improved in WHO FC after receiving medical therapy. For patients with IPAH response to therapy was better among patients who were younger, had higher WHO FC, had high diffusing capacity of the lung for carbon monoxide, had high mean pulmonary artery pressure, and had low PCO2.
“Our data suggest that PH-targeted drug therapy in patients with COPD and severe PH may improve exercise tolerance and WHO FC in a subgroup of patients and that patients with COPD and PH who respond to therapy may have a better prognosis than patients who do not show clinical improvement. These findings need to be explored further in prospective, randomized controlled clinical studies,” the authors concluded.
More research needed
In a related editorial, James R. Klinger, MD, a pulmonologist with Brown University, Providence, R.I., and the director of the Rhode Island Hospital Pulmonary Hypertension Center in East Providence, said there is a “keen interest” in treating PH in COPD despite a lack of consistency on whether treatment is effective in this patient population. About 80% of PH centers in the United States treat PH in COPD when they treat conditions like lung disease with PAH medication, he pointed out. However, he questioned whether current medications designed for PAH could improve pulmonary hemodynamics for PH in COPD.
“Reasons that the pathobiologic condition of PH-COPD may differ from PAH include the likely exposure of the pulmonary circulation to greater degrees of hypoxia and hypercapnia and the greater loss of alveolar capillaries associated with emphysema,” he said.
The study by Dr. Dario Vizza and colleagues is an attempt to evaluate treatment response for patients with PH and COPD “in a way that allows comparison with patients who have been treated with similar drugs for PAH,” Dr. Klinger said. He noted the study’s retrospective nature, lack of control group, and lack of information on lung disease severity could limit the findings.
“These limitations preclude recommendations for the routine treatment of patients with PH-COPD, but the findings suggest that, despite greater morbidity at baseline, patients with PH-COPD may be nearly as likely to benefit from PAH medications as patients with IPAH,” he said.
“What is needed now is well-designed randomized controlled studies to determine whether improved outcomes can be achieved in this population and which patients are most likely to benefit,” he concluded. “Simply put: How bad does PH need to be in patients with COPD before treatment is helpful, and how severe does COPD need to be before PH treatment is futile?”
The authors reported personal and institutional relationships in the form of grants, consultancies, advisory board memberships, speakers bureau appointments, honoraria, patents, grant and research funding, lectures, travel compensation, and steering committee positions for a variety of pharmaceutical companies, agencies, societies, journals, medical publishing companies, and other organizations. Dr. Klinger reported his institution receives grant support from Acceleron and United Therapeutics in the area of PH, and he has been an unpaid consultant for Bayer.
Patients with pulmonary hypertension (PH) as a complication of chronic obstructive pulmonary disease (COPD) have worse functional impairment and higher mortality, compared with patients who have idiopathic pulmonary arterial hypertension (IPAH).
Despite these factors, some patients with more severe PH in COPD may respond to treatment and show clinical improvement after treatment, according to recent research published in the journal CHEST®.
Carmine Dario Vizza, MD, of the pulmonary hypertension unit, department of cardiovascular and respiratory diseases at Sapienza University of Rome, and colleagues evaluated patients in the Comparative, Prospective Registry of Newly Initiated Therapies for Pulmonary Hypertension (COMPERA) database, enrolled up to August 2020, identifying 68 patients with moderate PH and COPD and 307 patients with severe PH and COPD. The researchers compared the PH and COPD groups with 307 patients who had idiopathic pulmonary arterial hypertension.
Overall, mostly older men made up the group of patients with moderate (50%; mean, 68.5 years) and severe PH in COPD (61%; mean 68.4 years), compared with those who had IPAH (37%; mean 61.7 years. Oral monotherapy for patients with PH and COPD was the main treatment, consisting of phosphodiesterase-5 inhibitors, while most patients with IPAH received endothelin receptor antagonists.
On functional tests, patients in the PH and COPD group tended to perform poorer on the 6-minute walking distance (6MWD) and World Health Organization functional class (WHO FC) than patients with IPAH. Specifically, among 42.7% of patients in both group for whom follow-up data were available, there was a similar frequency of improvement for 6MWD of 30 meters or more from baseline for both PH and COPD and IPAH groups (46.9% vs. 52.6%; P = .294), but there were significant differences between 6MWD between patients with moderate and severe PH and COPD (51.6% vs. 31.6%; P = .04). There was a nonsignificant improvement in WHO FC improvement of one or more classes for 65.6% of patients with PH and COPD and 58.3% of patients with IPAH with follow-up data available, with 28.5% of patients with PH in COPD improving compared with 35.8% of patients with IPAH (P = .078) and nonsignificant differences between moderate and severe PH and COPD (19.0% vs. 30.4%; P = .188).
Comparing outcomes
Follow-up data were available for 84% of patients with IPAH and 94% of patients with PH and COPD. Dr. Dario Vizza and colleagues found 45.7% of patients in the PH and COPD group and 24.9% of patients in the IPAH group died during follow-up, while 1.1% in the PH and COPD group and 1.5% of patients in the IPAH group underwent lung transplantations. For patients with moderate PH and COPD, 31.3% died and none underwent lung transplantation, while 49.0% of patients in the severe PH and COPD group died and 1.4% underwent lung transplantations.
Patients in the moderate PH and COPD group were more likely to discontinue treatment (10.9%), compared with patients with IPAH (6.6%) and patients with severe PH and COPD (5.2%). The most common reasons for discontinuations were tolerability and efficacy failure; the IPAH group had 63% of patients discontinue because of tolerability and 7% for efficacy failure, 47% of patients in the severe PH and COPD group discontinued because of tolerability and efficacy, and 29% discontinued treatment for tolerability and 57% for efficacy failure in the moderate and COPD group.
The researchers said male sex, low 6MWD, and high pulmonary vascular resistance at baseline were predictive of poorer outcomes for PH and COPD, but patients with more severe PH and COPD had better outcomes if they improved by 30 meters or more in 6MWD, or improved in WHO FC after receiving medical therapy. For patients with IPAH response to therapy was better among patients who were younger, had higher WHO FC, had high diffusing capacity of the lung for carbon monoxide, had high mean pulmonary artery pressure, and had low PCO2.
“Our data suggest that PH-targeted drug therapy in patients with COPD and severe PH may improve exercise tolerance and WHO FC in a subgroup of patients and that patients with COPD and PH who respond to therapy may have a better prognosis than patients who do not show clinical improvement. These findings need to be explored further in prospective, randomized controlled clinical studies,” the authors concluded.
More research needed
In a related editorial, James R. Klinger, MD, a pulmonologist with Brown University, Providence, R.I., and the director of the Rhode Island Hospital Pulmonary Hypertension Center in East Providence, said there is a “keen interest” in treating PH in COPD despite a lack of consistency on whether treatment is effective in this patient population. About 80% of PH centers in the United States treat PH in COPD when they treat conditions like lung disease with PAH medication, he pointed out. However, he questioned whether current medications designed for PAH could improve pulmonary hemodynamics for PH in COPD.
“Reasons that the pathobiologic condition of PH-COPD may differ from PAH include the likely exposure of the pulmonary circulation to greater degrees of hypoxia and hypercapnia and the greater loss of alveolar capillaries associated with emphysema,” he said.
The study by Dr. Dario Vizza and colleagues is an attempt to evaluate treatment response for patients with PH and COPD “in a way that allows comparison with patients who have been treated with similar drugs for PAH,” Dr. Klinger said. He noted the study’s retrospective nature, lack of control group, and lack of information on lung disease severity could limit the findings.
“These limitations preclude recommendations for the routine treatment of patients with PH-COPD, but the findings suggest that, despite greater morbidity at baseline, patients with PH-COPD may be nearly as likely to benefit from PAH medications as patients with IPAH,” he said.
“What is needed now is well-designed randomized controlled studies to determine whether improved outcomes can be achieved in this population and which patients are most likely to benefit,” he concluded. “Simply put: How bad does PH need to be in patients with COPD before treatment is helpful, and how severe does COPD need to be before PH treatment is futile?”
The authors reported personal and institutional relationships in the form of grants, consultancies, advisory board memberships, speakers bureau appointments, honoraria, patents, grant and research funding, lectures, travel compensation, and steering committee positions for a variety of pharmaceutical companies, agencies, societies, journals, medical publishing companies, and other organizations. Dr. Klinger reported his institution receives grant support from Acceleron and United Therapeutics in the area of PH, and he has been an unpaid consultant for Bayer.
FROM THE JOURNAL CHEST®
Use of point-of-care ultrasound (POCUS) for heart failure
Case
A 65-year-old woman presents to the emergency department with a chief complaint of shortness of breath for 3 days. Medical history is notable for moderate chronic obstructive pulmonary disorder, systolic heart failure with last known ejection fraction (EF) of 35% and type 2 diabetes complicated by hyperglycemia when on steroids. You are talking the case over with colleagues and they suggest point-of-care ultrasound (POCUS) would be useful in her case.
Brief overview of the issue
Once mainly used by ED and critical care physicians, POCUS is now a tool that many hospitalists are using at the bedside. POCUS differs from traditional comprehensive ultrasounds in the following ways: POCUS is designed to answer a specific clinical question (as opposed to evaluating all organs in a specific region), POCUS exams are performed by the clinician who is formulating the clinical question (as opposed to by a consultative service such as cardiology and radiology), and POCUS can evaluate multiple organ systems (such as by evaluating a patient’s heart, lungs, and inferior vena cava to determine the etiology of hypoxia).
Hospitalist use of POCUS may include guiding procedures, aiding in diagnosis, and assessing effectiveness of treatment. Many high-quality studies have been published that support the use of POCUS and have proven that POCUS can decrease medical errors, help reach diagnoses in a more expedited fashion, and complement or replace more advanced imaging.
A challenge of POCUS is that it is user dependent and there are no established standards for hospitalists in POCUS training. As the Society of Hospital Medicine position statement on POCUS points out, there is a significant difference between skill levels required to obtain a certificate of completion for POCUS training and a certificate of competency in POCUS. Therefore, it is recommended hospitalists work with local credentialing committees to delineate the requirements for POCUS use.
Overview of the data
POCUS for initial assessment and diagnosis of heart failure (HF)
Use of POCUS in cases of suspected HF includes examination of the heart, lungs, and inferior vena cava (IVC). Cardiac ultrasound provides an estimated ejection fraction. Lung ultrasound (LUS) functions to examine for B lines and pleural effusions. The presence of more than three B lines per thoracic zone bilaterally suggests cardiogenic pulmonary edema. Scanning the IVC provides a noninvasive way to assess volume status and is especially helpful when body habitus prevents accurate assessment of jugular venous pressure.
Several studies have addressed the utility of bedside ultrasound in the initial assessment or diagnosis of acute decompensated heart failure (ADHF) in patients presenting with dyspnea in emergency or inpatient settings. Positive B lines are a useful finding, with high sensitivities, high specificities, and positive likelihood ratios. One large multicenter prospective study found LUS to have a sensitivity of 90.5%, specificity of 93.5%, and positive and negative LRs of 14.0 and 0.10, respectively.1 Another large multicenter prospective cohort study showed that LUS was more sensitive and more specific than chest x-ray (CXR) and brain natriuretic peptide in detecting ADHF.2 Additional POCUS findings that have shown relatively high sensitivities and specificities in the initial diagnosis of ADHF include pleural effusion, reduced left ventricular ejection fraction (LVEF), increased left ventricular end-diastolic dimension, and jugular venous distention.
Data also exists on assessments of ADHF using combinations of POCUS findings; for example, lung and cardiac ultrasound (LuCUS) protocols include an evaluation for B lines, assessment of IVC size and collapsibility, and determination of LVEF, although this has mainly been examined in ED patients. For patients who presented to the ED with undifferentiated dyspnea, one such study showed a specificity of 100% when a LuCUS protocol was used to diagnose ADHF while another study showed that the use of a LuCUS protocol changed management in 47% of patients.3,4 Of note, although each LuCUS protocol integrated the use of lung findings, IVC collapsibility, and LVEF, the exact protocols varied by institution. Finally, it has been established in multiple studies that LUS used in addition to standard workup including history and physical, labs, and electrocardiogram has been shown to increase diagnostic accuracy.2,5
Using POCUS to guide diuretic therapy in HF
To date, there have been multiple small studies published on the utility of daily POCUS in hospitalized patients with ADHF to help assess response to treatment and guide diuresis by looking for reduction in B lines on LUS or a change in IVC size or collapsibility. Volpicelli and colleagues showed that daily LUS was at least as good as daily CXR in monitoring response to therapy.6 Similarly, Mozzini and colleagues performed a randomized controlled trial of 120 patients admitted for ADHF who were randomized to a CXR group (who had a CXR performed on admission and discharge) and a LUS group (which was performed at admission, 24 hours, 48 hours, 72 hours, and discharge).7 This study found that the LUS group underwent a significantly higher number of diuretic dose adjustments as compared with the CXR group (P < .001) and had a modest improvement in LOS, compared with the CXR group. Specifically, median LOS was 8 days in CXR group (range, 4-17 days) and 7 days in the LUS group (range, 3-10 days; P < .001).
The impact of POCUS on length of stay (LOS) and readmissions
There is increasing data that POCUS can have meaningful impacts on patient-centered outcomes (morbidity, mortality, and readmission) while exposing patients to minimal discomfort, no venipuncture, and no radiation exposure. First, multiple studies looked at whether performing focused cardiac US of the IVC as a marker of volume status could predict readmission in patients hospitalized for ADHF.8,9 Both of these trials showed that plethoric, noncollapsible IVC at discharge were statistically significant predictors of readmission. In fact, Goonewardena and colleagues demonstrated that patients who required readmission had an enlarged IVC at discharge nearly 3 times more frequently (21% vs. 61%, P < .001) and abnormal IVC collapsibility 1.5 times more frequently (41% vs. 71%, P = .01) as compared with patients who remained out of the hospital.9
Similarly, a subsequent trial looked at whether IVC size on admission was of prognostic importance in patients hospitalized for ADHF and showed that admission IVC diameter was an independent predictor of both 90-day mortality (hazard ratio, 5.88; 95% confidence interval, 1.21-28.10; P = .025) and 90-day readmission (HR, 3.20; 95% CI, 1.24-8.21; P = .016).10 Additionally, LUS heart failure assessment for pulmonary congestion by counting B lines also showed that having more than 15 B lines prior to discharge was an independent predictor of readmission for ADHF at 6 months (HR, 11.74; 95% CI, 1.30-106.16).11
A challenge of POCUS: Obtaining competency
As previously noted, there are not yet any established standards for training and assessing hospitalists in POCUS. The SHM Position Statement on POCUS recommends the following criteria for training5: the training environment should be similar to the location in which the trainee will practice, training and feedback should occur in real time, the trainee should be taught specific applications of POCUS (such as cardiac US, LUS, and IVC US) as each application comes with unique skills and knowledge, clinical competence must be achieved and demonstrated, and continued education and feedback are necessary once competence is obtained.12 SHM recommends residency-based training pathways, training through a local or national program such as the SHM POCUS certificate program, or training through other medical societies for hospitalists already in practice.
Application of the data to our original case
Targeted POCUS using the LuCUS protocol is performed and reveals three B lines in two lung zones bilaterally, moderate bilateral pleural effusions, EF 20%, and a noncollapsible IVC leading to a diagnosis of ADHF. Her ADHF is treated with intravenous diuresis. She is continued on her chronic maintenance chronic obstructive pulmonary disorder regimen but does not receive steroids, avoiding hyperglycemia that has complicated prior admissions. Over the next few days her respiratory and cardiac status is monitored using POCUS to assess her response to therapy and titrate her diuretics to her true dry weight, which was several pounds lower than her previously assumed dry weight. At discharge she is instructed to use the new dry weight which may avoid readmissions for HF.
Bottom line
POCUS improves diagnostic accuracy and facilitates volume assessment and management in acute decompensated heart failure.
Dr. Farber is a medical instructor at Duke University and hospitalist at Duke Regional Hospital, both in Durham, N.C. Dr. Marcantonio is a medical instructor in the department of internal medicine and department of pediatrics at Duke University and hospitalist at Duke University Hospital and Duke Regional Hospital. Dr. Stafford and Dr. Brooks are assistant professors of medicine and hospitalists at Duke Regional Hospital. Dr. Wachter is associate medical director at Duke Regional Hospital and assistant professor at Duke University. Dr. Menon is a hospitalist at Duke University. Dr. Sharma is associate medical director for clinical education at Duke Regional Hospital and associate professor of medicine at Duke University.
References
1. Pivetta E et al. Lung ultrasound integrated with clinical assessment for the diagnosis of acute decompensated heart failure in the emergency department: A randomized controlled trial. Eur J Heart Fail. 2019 Jun;21(6):754-66. doi: 10.1002/ejhf.1379.
2. Pivetta E et al. Lung ultrasound-implemented diagnosis of acute decompensated heart failure in the ED: A SIMEU multicenter study. Chest. 2015;148(1):202-10. doi: 10.1378/chest.14-2608.
3. Anderson KL et al. Diagnosing heart failure among acutely dyspneic patients with cardiac, inferior vena cava, and lung ultrasonography. Am J Emerg Med. 2013;31:1208-14. doi: 10.1016/j.ajem.2013.05.007.
4. Russell FM et al. Diagnosing acute heart failure in patients with undifferentiated dyspnea: A lung and cardiac ultrasound (LuCUS) protocol. Acad Emerg Med. 2015;22(2):182-91. doi:10.1111/acem.12570.
5. Maw AM et al. Diagnostic accuracy of point-of-care lung ultrasonography and chest radiography in adults with symptoms suggestive of acute decompensated heart failure: A systematic review and meta-analysis. JAMA Netw Open. 2019 Mar 1;2(3):e190703. doi:10.1001/jamanetworkopen.2019.0703.
6. Volpicelli G et al. Bedside ultrasound of the lung for the monitoring of acute decompensated heart failure. Am J Emerg Med. 2008 Jun;26(5):585-91. doi:10.1016/j.ajem.2007.09.014.
7. Mozzini C et al. Lung ultrasound in internal medicine efficiently drives the management of patients with heart failure and speeds up the discharge time. Intern Emerg Med. 2018 Jan;13(1):27-33. doi: 10.1007/s11739-017-1738-1.
8. Laffin LJ et al. Focused cardiac ultrasound as a predictor of readmission in acute decompensated heart failure. Int J Cardiovasc Imaging. 2018;34(7):1075-9. doi:10.1007/s10554-018-1317-1.
9. Goonewardena SN et al. Comparison of hand-carried ultrasound assessment of the inferior vena cava and N-terminal pro-brain natriuretic peptide for predicting readmission after hospitalization for acute decompensated heart failure. JACC Cardiovasc Imaging. 2008;1(5):595-601. doi:10.1016/j.jcmg.2008.06.005.
10. Cubo-Romano P et al. Admission inferior vena cava measurements are associated with mortality after hospitalization for acute decompensated heart failure. J Hosp Med. 2016 Nov;11(11):778-84. doi: 10.1002/jhm.2620.
11. Gargani L et al. Persistent pulmonary congestion before discharge predicts rehospitalization in heart failure: A lung ultrasound study. Cardiovasc Ultrasound. 2015 Sep 4;13:40. doi: 10.1186/s12947-015-0033-4.
12. Soni NJ et al. Point-of-care ultrasound for hospitalists: A Position Statement of the Society of Hospital Medicine. J Hosp Med. 2019 Jan 2;14:E1-6. doi: 10.12788/jhm.3079.
Key points
- Studies have found POCUS improves the diagnosis of acute decompensated heart failure in patients presenting with dyspnea.
- Daily evaluation with POCUS has decreased length of stay in acute decompensated heart failure.
- Credentialing requirements for hospitalists to use POCUS for clinical care vary by hospital.
Additional reading
Maw AM and Soni NJ. Annals for hospitalists inpatient notes – why should hospitalists use point-of-care ultrasound? Ann Intern Med. 2018 Apr 17;168(8):HO2-HO3. doi: 10.7326/M18-0367.
Lewiss RE. “The ultrasound looked fine”: Point of care ultrasound and patient safety. AHRQ’s Patient Safety Network. WebM&M: Case Studies. 2018 Jul 1. https://psnet.ahrq.gov/web-mm/ultrasound-looked-fine-point-care-ultrasound-and-patient-safety.
Quiz: Testing your POCUS knowledge
POCUS is increasingly prevalent in hospital medicine, but use varies among different disease processes. Which organ system ultrasound or lab test would be most helpful in the following scenario?
An acutely dyspneic patient with no past medical history presents to the ED. Chest x-ray is equivocal. Of the following, which study best confirms a diagnosis of acute decompensated heart failure?
A. Brain natriuretic peptide
B. Point-of-care cardiac ultrasound
C. Point-of-care lung ultrasound
D. Point-of-care inferior vena cava ultrasound
Answer
C. Point-of-care lung ultrasound
Multiple studies, including three systematic reviews, have shown that point-of-care lung ultrasound has high sensitivity and specificity to evaluate for B lines as a marker for cardiogenic pulmonary edema. Point-of-care ultrasound of ejection fraction and inferior vena cava have not been evaluated by systematic review although one randomized, controlled trial showed that an EF less than 45% had 74% specificity and 77% sensitivity and IVC collapsibility index less than 20% had an 86% specificity and 52% sensitivity for detection of acute decompensated heart failure. This same study showed that the combination of cardiac, lung, and IVC point-of-care ultrasound had 100% specificity for diagnosing acute decompensated heart failure. In the future, health care providers could rely on this multiorgan evaluation with point-of-care ultrasound to confirm a diagnosis of acute decompensated heart failure in a dyspneic patient.
Case
A 65-year-old woman presents to the emergency department with a chief complaint of shortness of breath for 3 days. Medical history is notable for moderate chronic obstructive pulmonary disorder, systolic heart failure with last known ejection fraction (EF) of 35% and type 2 diabetes complicated by hyperglycemia when on steroids. You are talking the case over with colleagues and they suggest point-of-care ultrasound (POCUS) would be useful in her case.
Brief overview of the issue
Once mainly used by ED and critical care physicians, POCUS is now a tool that many hospitalists are using at the bedside. POCUS differs from traditional comprehensive ultrasounds in the following ways: POCUS is designed to answer a specific clinical question (as opposed to evaluating all organs in a specific region), POCUS exams are performed by the clinician who is formulating the clinical question (as opposed to by a consultative service such as cardiology and radiology), and POCUS can evaluate multiple organ systems (such as by evaluating a patient’s heart, lungs, and inferior vena cava to determine the etiology of hypoxia).
Hospitalist use of POCUS may include guiding procedures, aiding in diagnosis, and assessing effectiveness of treatment. Many high-quality studies have been published that support the use of POCUS and have proven that POCUS can decrease medical errors, help reach diagnoses in a more expedited fashion, and complement or replace more advanced imaging.
A challenge of POCUS is that it is user dependent and there are no established standards for hospitalists in POCUS training. As the Society of Hospital Medicine position statement on POCUS points out, there is a significant difference between skill levels required to obtain a certificate of completion for POCUS training and a certificate of competency in POCUS. Therefore, it is recommended hospitalists work with local credentialing committees to delineate the requirements for POCUS use.
Overview of the data
POCUS for initial assessment and diagnosis of heart failure (HF)
Use of POCUS in cases of suspected HF includes examination of the heart, lungs, and inferior vena cava (IVC). Cardiac ultrasound provides an estimated ejection fraction. Lung ultrasound (LUS) functions to examine for B lines and pleural effusions. The presence of more than three B lines per thoracic zone bilaterally suggests cardiogenic pulmonary edema. Scanning the IVC provides a noninvasive way to assess volume status and is especially helpful when body habitus prevents accurate assessment of jugular venous pressure.
Several studies have addressed the utility of bedside ultrasound in the initial assessment or diagnosis of acute decompensated heart failure (ADHF) in patients presenting with dyspnea in emergency or inpatient settings. Positive B lines are a useful finding, with high sensitivities, high specificities, and positive likelihood ratios. One large multicenter prospective study found LUS to have a sensitivity of 90.5%, specificity of 93.5%, and positive and negative LRs of 14.0 and 0.10, respectively.1 Another large multicenter prospective cohort study showed that LUS was more sensitive and more specific than chest x-ray (CXR) and brain natriuretic peptide in detecting ADHF.2 Additional POCUS findings that have shown relatively high sensitivities and specificities in the initial diagnosis of ADHF include pleural effusion, reduced left ventricular ejection fraction (LVEF), increased left ventricular end-diastolic dimension, and jugular venous distention.
Data also exists on assessments of ADHF using combinations of POCUS findings; for example, lung and cardiac ultrasound (LuCUS) protocols include an evaluation for B lines, assessment of IVC size and collapsibility, and determination of LVEF, although this has mainly been examined in ED patients. For patients who presented to the ED with undifferentiated dyspnea, one such study showed a specificity of 100% when a LuCUS protocol was used to diagnose ADHF while another study showed that the use of a LuCUS protocol changed management in 47% of patients.3,4 Of note, although each LuCUS protocol integrated the use of lung findings, IVC collapsibility, and LVEF, the exact protocols varied by institution. Finally, it has been established in multiple studies that LUS used in addition to standard workup including history and physical, labs, and electrocardiogram has been shown to increase diagnostic accuracy.2,5
Using POCUS to guide diuretic therapy in HF
To date, there have been multiple small studies published on the utility of daily POCUS in hospitalized patients with ADHF to help assess response to treatment and guide diuresis by looking for reduction in B lines on LUS or a change in IVC size or collapsibility. Volpicelli and colleagues showed that daily LUS was at least as good as daily CXR in monitoring response to therapy.6 Similarly, Mozzini and colleagues performed a randomized controlled trial of 120 patients admitted for ADHF who were randomized to a CXR group (who had a CXR performed on admission and discharge) and a LUS group (which was performed at admission, 24 hours, 48 hours, 72 hours, and discharge).7 This study found that the LUS group underwent a significantly higher number of diuretic dose adjustments as compared with the CXR group (P < .001) and had a modest improvement in LOS, compared with the CXR group. Specifically, median LOS was 8 days in CXR group (range, 4-17 days) and 7 days in the LUS group (range, 3-10 days; P < .001).
The impact of POCUS on length of stay (LOS) and readmissions
There is increasing data that POCUS can have meaningful impacts on patient-centered outcomes (morbidity, mortality, and readmission) while exposing patients to minimal discomfort, no venipuncture, and no radiation exposure. First, multiple studies looked at whether performing focused cardiac US of the IVC as a marker of volume status could predict readmission in patients hospitalized for ADHF.8,9 Both of these trials showed that plethoric, noncollapsible IVC at discharge were statistically significant predictors of readmission. In fact, Goonewardena and colleagues demonstrated that patients who required readmission had an enlarged IVC at discharge nearly 3 times more frequently (21% vs. 61%, P < .001) and abnormal IVC collapsibility 1.5 times more frequently (41% vs. 71%, P = .01) as compared with patients who remained out of the hospital.9
Similarly, a subsequent trial looked at whether IVC size on admission was of prognostic importance in patients hospitalized for ADHF and showed that admission IVC diameter was an independent predictor of both 90-day mortality (hazard ratio, 5.88; 95% confidence interval, 1.21-28.10; P = .025) and 90-day readmission (HR, 3.20; 95% CI, 1.24-8.21; P = .016).10 Additionally, LUS heart failure assessment for pulmonary congestion by counting B lines also showed that having more than 15 B lines prior to discharge was an independent predictor of readmission for ADHF at 6 months (HR, 11.74; 95% CI, 1.30-106.16).11
A challenge of POCUS: Obtaining competency
As previously noted, there are not yet any established standards for training and assessing hospitalists in POCUS. The SHM Position Statement on POCUS recommends the following criteria for training5: the training environment should be similar to the location in which the trainee will practice, training and feedback should occur in real time, the trainee should be taught specific applications of POCUS (such as cardiac US, LUS, and IVC US) as each application comes with unique skills and knowledge, clinical competence must be achieved and demonstrated, and continued education and feedback are necessary once competence is obtained.12 SHM recommends residency-based training pathways, training through a local or national program such as the SHM POCUS certificate program, or training through other medical societies for hospitalists already in practice.
Application of the data to our original case
Targeted POCUS using the LuCUS protocol is performed and reveals three B lines in two lung zones bilaterally, moderate bilateral pleural effusions, EF 20%, and a noncollapsible IVC leading to a diagnosis of ADHF. Her ADHF is treated with intravenous diuresis. She is continued on her chronic maintenance chronic obstructive pulmonary disorder regimen but does not receive steroids, avoiding hyperglycemia that has complicated prior admissions. Over the next few days her respiratory and cardiac status is monitored using POCUS to assess her response to therapy and titrate her diuretics to her true dry weight, which was several pounds lower than her previously assumed dry weight. At discharge she is instructed to use the new dry weight which may avoid readmissions for HF.
Bottom line
POCUS improves diagnostic accuracy and facilitates volume assessment and management in acute decompensated heart failure.
Dr. Farber is a medical instructor at Duke University and hospitalist at Duke Regional Hospital, both in Durham, N.C. Dr. Marcantonio is a medical instructor in the department of internal medicine and department of pediatrics at Duke University and hospitalist at Duke University Hospital and Duke Regional Hospital. Dr. Stafford and Dr. Brooks are assistant professors of medicine and hospitalists at Duke Regional Hospital. Dr. Wachter is associate medical director at Duke Regional Hospital and assistant professor at Duke University. Dr. Menon is a hospitalist at Duke University. Dr. Sharma is associate medical director for clinical education at Duke Regional Hospital and associate professor of medicine at Duke University.
References
1. Pivetta E et al. Lung ultrasound integrated with clinical assessment for the diagnosis of acute decompensated heart failure in the emergency department: A randomized controlled trial. Eur J Heart Fail. 2019 Jun;21(6):754-66. doi: 10.1002/ejhf.1379.
2. Pivetta E et al. Lung ultrasound-implemented diagnosis of acute decompensated heart failure in the ED: A SIMEU multicenter study. Chest. 2015;148(1):202-10. doi: 10.1378/chest.14-2608.
3. Anderson KL et al. Diagnosing heart failure among acutely dyspneic patients with cardiac, inferior vena cava, and lung ultrasonography. Am J Emerg Med. 2013;31:1208-14. doi: 10.1016/j.ajem.2013.05.007.
4. Russell FM et al. Diagnosing acute heart failure in patients with undifferentiated dyspnea: A lung and cardiac ultrasound (LuCUS) protocol. Acad Emerg Med. 2015;22(2):182-91. doi:10.1111/acem.12570.
5. Maw AM et al. Diagnostic accuracy of point-of-care lung ultrasonography and chest radiography in adults with symptoms suggestive of acute decompensated heart failure: A systematic review and meta-analysis. JAMA Netw Open. 2019 Mar 1;2(3):e190703. doi:10.1001/jamanetworkopen.2019.0703.
6. Volpicelli G et al. Bedside ultrasound of the lung for the monitoring of acute decompensated heart failure. Am J Emerg Med. 2008 Jun;26(5):585-91. doi:10.1016/j.ajem.2007.09.014.
7. Mozzini C et al. Lung ultrasound in internal medicine efficiently drives the management of patients with heart failure and speeds up the discharge time. Intern Emerg Med. 2018 Jan;13(1):27-33. doi: 10.1007/s11739-017-1738-1.
8. Laffin LJ et al. Focused cardiac ultrasound as a predictor of readmission in acute decompensated heart failure. Int J Cardiovasc Imaging. 2018;34(7):1075-9. doi:10.1007/s10554-018-1317-1.
9. Goonewardena SN et al. Comparison of hand-carried ultrasound assessment of the inferior vena cava and N-terminal pro-brain natriuretic peptide for predicting readmission after hospitalization for acute decompensated heart failure. JACC Cardiovasc Imaging. 2008;1(5):595-601. doi:10.1016/j.jcmg.2008.06.005.
10. Cubo-Romano P et al. Admission inferior vena cava measurements are associated with mortality after hospitalization for acute decompensated heart failure. J Hosp Med. 2016 Nov;11(11):778-84. doi: 10.1002/jhm.2620.
11. Gargani L et al. Persistent pulmonary congestion before discharge predicts rehospitalization in heart failure: A lung ultrasound study. Cardiovasc Ultrasound. 2015 Sep 4;13:40. doi: 10.1186/s12947-015-0033-4.
12. Soni NJ et al. Point-of-care ultrasound for hospitalists: A Position Statement of the Society of Hospital Medicine. J Hosp Med. 2019 Jan 2;14:E1-6. doi: 10.12788/jhm.3079.
Key points
- Studies have found POCUS improves the diagnosis of acute decompensated heart failure in patients presenting with dyspnea.
- Daily evaluation with POCUS has decreased length of stay in acute decompensated heart failure.
- Credentialing requirements for hospitalists to use POCUS for clinical care vary by hospital.
Additional reading
Maw AM and Soni NJ. Annals for hospitalists inpatient notes – why should hospitalists use point-of-care ultrasound? Ann Intern Med. 2018 Apr 17;168(8):HO2-HO3. doi: 10.7326/M18-0367.
Lewiss RE. “The ultrasound looked fine”: Point of care ultrasound and patient safety. AHRQ’s Patient Safety Network. WebM&M: Case Studies. 2018 Jul 1. https://psnet.ahrq.gov/web-mm/ultrasound-looked-fine-point-care-ultrasound-and-patient-safety.
Quiz: Testing your POCUS knowledge
POCUS is increasingly prevalent in hospital medicine, but use varies among different disease processes. Which organ system ultrasound or lab test would be most helpful in the following scenario?
An acutely dyspneic patient with no past medical history presents to the ED. Chest x-ray is equivocal. Of the following, which study best confirms a diagnosis of acute decompensated heart failure?
A. Brain natriuretic peptide
B. Point-of-care cardiac ultrasound
C. Point-of-care lung ultrasound
D. Point-of-care inferior vena cava ultrasound
Answer
C. Point-of-care lung ultrasound
Multiple studies, including three systematic reviews, have shown that point-of-care lung ultrasound has high sensitivity and specificity to evaluate for B lines as a marker for cardiogenic pulmonary edema. Point-of-care ultrasound of ejection fraction and inferior vena cava have not been evaluated by systematic review although one randomized, controlled trial showed that an EF less than 45% had 74% specificity and 77% sensitivity and IVC collapsibility index less than 20% had an 86% specificity and 52% sensitivity for detection of acute decompensated heart failure. This same study showed that the combination of cardiac, lung, and IVC point-of-care ultrasound had 100% specificity for diagnosing acute decompensated heart failure. In the future, health care providers could rely on this multiorgan evaluation with point-of-care ultrasound to confirm a diagnosis of acute decompensated heart failure in a dyspneic patient.
Case
A 65-year-old woman presents to the emergency department with a chief complaint of shortness of breath for 3 days. Medical history is notable for moderate chronic obstructive pulmonary disorder, systolic heart failure with last known ejection fraction (EF) of 35% and type 2 diabetes complicated by hyperglycemia when on steroids. You are talking the case over with colleagues and they suggest point-of-care ultrasound (POCUS) would be useful in her case.
Brief overview of the issue
Once mainly used by ED and critical care physicians, POCUS is now a tool that many hospitalists are using at the bedside. POCUS differs from traditional comprehensive ultrasounds in the following ways: POCUS is designed to answer a specific clinical question (as opposed to evaluating all organs in a specific region), POCUS exams are performed by the clinician who is formulating the clinical question (as opposed to by a consultative service such as cardiology and radiology), and POCUS can evaluate multiple organ systems (such as by evaluating a patient’s heart, lungs, and inferior vena cava to determine the etiology of hypoxia).
Hospitalist use of POCUS may include guiding procedures, aiding in diagnosis, and assessing effectiveness of treatment. Many high-quality studies have been published that support the use of POCUS and have proven that POCUS can decrease medical errors, help reach diagnoses in a more expedited fashion, and complement or replace more advanced imaging.
A challenge of POCUS is that it is user dependent and there are no established standards for hospitalists in POCUS training. As the Society of Hospital Medicine position statement on POCUS points out, there is a significant difference between skill levels required to obtain a certificate of completion for POCUS training and a certificate of competency in POCUS. Therefore, it is recommended hospitalists work with local credentialing committees to delineate the requirements for POCUS use.
Overview of the data
POCUS for initial assessment and diagnosis of heart failure (HF)
Use of POCUS in cases of suspected HF includes examination of the heart, lungs, and inferior vena cava (IVC). Cardiac ultrasound provides an estimated ejection fraction. Lung ultrasound (LUS) functions to examine for B lines and pleural effusions. The presence of more than three B lines per thoracic zone bilaterally suggests cardiogenic pulmonary edema. Scanning the IVC provides a noninvasive way to assess volume status and is especially helpful when body habitus prevents accurate assessment of jugular venous pressure.
Several studies have addressed the utility of bedside ultrasound in the initial assessment or diagnosis of acute decompensated heart failure (ADHF) in patients presenting with dyspnea in emergency or inpatient settings. Positive B lines are a useful finding, with high sensitivities, high specificities, and positive likelihood ratios. One large multicenter prospective study found LUS to have a sensitivity of 90.5%, specificity of 93.5%, and positive and negative LRs of 14.0 and 0.10, respectively.1 Another large multicenter prospective cohort study showed that LUS was more sensitive and more specific than chest x-ray (CXR) and brain natriuretic peptide in detecting ADHF.2 Additional POCUS findings that have shown relatively high sensitivities and specificities in the initial diagnosis of ADHF include pleural effusion, reduced left ventricular ejection fraction (LVEF), increased left ventricular end-diastolic dimension, and jugular venous distention.
Data also exists on assessments of ADHF using combinations of POCUS findings; for example, lung and cardiac ultrasound (LuCUS) protocols include an evaluation for B lines, assessment of IVC size and collapsibility, and determination of LVEF, although this has mainly been examined in ED patients. For patients who presented to the ED with undifferentiated dyspnea, one such study showed a specificity of 100% when a LuCUS protocol was used to diagnose ADHF while another study showed that the use of a LuCUS protocol changed management in 47% of patients.3,4 Of note, although each LuCUS protocol integrated the use of lung findings, IVC collapsibility, and LVEF, the exact protocols varied by institution. Finally, it has been established in multiple studies that LUS used in addition to standard workup including history and physical, labs, and electrocardiogram has been shown to increase diagnostic accuracy.2,5
Using POCUS to guide diuretic therapy in HF
To date, there have been multiple small studies published on the utility of daily POCUS in hospitalized patients with ADHF to help assess response to treatment and guide diuresis by looking for reduction in B lines on LUS or a change in IVC size or collapsibility. Volpicelli and colleagues showed that daily LUS was at least as good as daily CXR in monitoring response to therapy.6 Similarly, Mozzini and colleagues performed a randomized controlled trial of 120 patients admitted for ADHF who were randomized to a CXR group (who had a CXR performed on admission and discharge) and a LUS group (which was performed at admission, 24 hours, 48 hours, 72 hours, and discharge).7 This study found that the LUS group underwent a significantly higher number of diuretic dose adjustments as compared with the CXR group (P < .001) and had a modest improvement in LOS, compared with the CXR group. Specifically, median LOS was 8 days in CXR group (range, 4-17 days) and 7 days in the LUS group (range, 3-10 days; P < .001).
The impact of POCUS on length of stay (LOS) and readmissions
There is increasing data that POCUS can have meaningful impacts on patient-centered outcomes (morbidity, mortality, and readmission) while exposing patients to minimal discomfort, no venipuncture, and no radiation exposure. First, multiple studies looked at whether performing focused cardiac US of the IVC as a marker of volume status could predict readmission in patients hospitalized for ADHF.8,9 Both of these trials showed that plethoric, noncollapsible IVC at discharge were statistically significant predictors of readmission. In fact, Goonewardena and colleagues demonstrated that patients who required readmission had an enlarged IVC at discharge nearly 3 times more frequently (21% vs. 61%, P < .001) and abnormal IVC collapsibility 1.5 times more frequently (41% vs. 71%, P = .01) as compared with patients who remained out of the hospital.9
Similarly, a subsequent trial looked at whether IVC size on admission was of prognostic importance in patients hospitalized for ADHF and showed that admission IVC diameter was an independent predictor of both 90-day mortality (hazard ratio, 5.88; 95% confidence interval, 1.21-28.10; P = .025) and 90-day readmission (HR, 3.20; 95% CI, 1.24-8.21; P = .016).10 Additionally, LUS heart failure assessment for pulmonary congestion by counting B lines also showed that having more than 15 B lines prior to discharge was an independent predictor of readmission for ADHF at 6 months (HR, 11.74; 95% CI, 1.30-106.16).11
A challenge of POCUS: Obtaining competency
As previously noted, there are not yet any established standards for training and assessing hospitalists in POCUS. The SHM Position Statement on POCUS recommends the following criteria for training5: the training environment should be similar to the location in which the trainee will practice, training and feedback should occur in real time, the trainee should be taught specific applications of POCUS (such as cardiac US, LUS, and IVC US) as each application comes with unique skills and knowledge, clinical competence must be achieved and demonstrated, and continued education and feedback are necessary once competence is obtained.12 SHM recommends residency-based training pathways, training through a local or national program such as the SHM POCUS certificate program, or training through other medical societies for hospitalists already in practice.
Application of the data to our original case
Targeted POCUS using the LuCUS protocol is performed and reveals three B lines in two lung zones bilaterally, moderate bilateral pleural effusions, EF 20%, and a noncollapsible IVC leading to a diagnosis of ADHF. Her ADHF is treated with intravenous diuresis. She is continued on her chronic maintenance chronic obstructive pulmonary disorder regimen but does not receive steroids, avoiding hyperglycemia that has complicated prior admissions. Over the next few days her respiratory and cardiac status is monitored using POCUS to assess her response to therapy and titrate her diuretics to her true dry weight, which was several pounds lower than her previously assumed dry weight. At discharge she is instructed to use the new dry weight which may avoid readmissions for HF.
Bottom line
POCUS improves diagnostic accuracy and facilitates volume assessment and management in acute decompensated heart failure.
Dr. Farber is a medical instructor at Duke University and hospitalist at Duke Regional Hospital, both in Durham, N.C. Dr. Marcantonio is a medical instructor in the department of internal medicine and department of pediatrics at Duke University and hospitalist at Duke University Hospital and Duke Regional Hospital. Dr. Stafford and Dr. Brooks are assistant professors of medicine and hospitalists at Duke Regional Hospital. Dr. Wachter is associate medical director at Duke Regional Hospital and assistant professor at Duke University. Dr. Menon is a hospitalist at Duke University. Dr. Sharma is associate medical director for clinical education at Duke Regional Hospital and associate professor of medicine at Duke University.
References
1. Pivetta E et al. Lung ultrasound integrated with clinical assessment for the diagnosis of acute decompensated heart failure in the emergency department: A randomized controlled trial. Eur J Heart Fail. 2019 Jun;21(6):754-66. doi: 10.1002/ejhf.1379.
2. Pivetta E et al. Lung ultrasound-implemented diagnosis of acute decompensated heart failure in the ED: A SIMEU multicenter study. Chest. 2015;148(1):202-10. doi: 10.1378/chest.14-2608.
3. Anderson KL et al. Diagnosing heart failure among acutely dyspneic patients with cardiac, inferior vena cava, and lung ultrasonography. Am J Emerg Med. 2013;31:1208-14. doi: 10.1016/j.ajem.2013.05.007.
4. Russell FM et al. Diagnosing acute heart failure in patients with undifferentiated dyspnea: A lung and cardiac ultrasound (LuCUS) protocol. Acad Emerg Med. 2015;22(2):182-91. doi:10.1111/acem.12570.
5. Maw AM et al. Diagnostic accuracy of point-of-care lung ultrasonography and chest radiography in adults with symptoms suggestive of acute decompensated heart failure: A systematic review and meta-analysis. JAMA Netw Open. 2019 Mar 1;2(3):e190703. doi:10.1001/jamanetworkopen.2019.0703.
6. Volpicelli G et al. Bedside ultrasound of the lung for the monitoring of acute decompensated heart failure. Am J Emerg Med. 2008 Jun;26(5):585-91. doi:10.1016/j.ajem.2007.09.014.
7. Mozzini C et al. Lung ultrasound in internal medicine efficiently drives the management of patients with heart failure and speeds up the discharge time. Intern Emerg Med. 2018 Jan;13(1):27-33. doi: 10.1007/s11739-017-1738-1.
8. Laffin LJ et al. Focused cardiac ultrasound as a predictor of readmission in acute decompensated heart failure. Int J Cardiovasc Imaging. 2018;34(7):1075-9. doi:10.1007/s10554-018-1317-1.
9. Goonewardena SN et al. Comparison of hand-carried ultrasound assessment of the inferior vena cava and N-terminal pro-brain natriuretic peptide for predicting readmission after hospitalization for acute decompensated heart failure. JACC Cardiovasc Imaging. 2008;1(5):595-601. doi:10.1016/j.jcmg.2008.06.005.
10. Cubo-Romano P et al. Admission inferior vena cava measurements are associated with mortality after hospitalization for acute decompensated heart failure. J Hosp Med. 2016 Nov;11(11):778-84. doi: 10.1002/jhm.2620.
11. Gargani L et al. Persistent pulmonary congestion before discharge predicts rehospitalization in heart failure: A lung ultrasound study. Cardiovasc Ultrasound. 2015 Sep 4;13:40. doi: 10.1186/s12947-015-0033-4.
12. Soni NJ et al. Point-of-care ultrasound for hospitalists: A Position Statement of the Society of Hospital Medicine. J Hosp Med. 2019 Jan 2;14:E1-6. doi: 10.12788/jhm.3079.
Key points
- Studies have found POCUS improves the diagnosis of acute decompensated heart failure in patients presenting with dyspnea.
- Daily evaluation with POCUS has decreased length of stay in acute decompensated heart failure.
- Credentialing requirements for hospitalists to use POCUS for clinical care vary by hospital.
Additional reading
Maw AM and Soni NJ. Annals for hospitalists inpatient notes – why should hospitalists use point-of-care ultrasound? Ann Intern Med. 2018 Apr 17;168(8):HO2-HO3. doi: 10.7326/M18-0367.
Lewiss RE. “The ultrasound looked fine”: Point of care ultrasound and patient safety. AHRQ’s Patient Safety Network. WebM&M: Case Studies. 2018 Jul 1. https://psnet.ahrq.gov/web-mm/ultrasound-looked-fine-point-care-ultrasound-and-patient-safety.
Quiz: Testing your POCUS knowledge
POCUS is increasingly prevalent in hospital medicine, but use varies among different disease processes. Which organ system ultrasound or lab test would be most helpful in the following scenario?
An acutely dyspneic patient with no past medical history presents to the ED. Chest x-ray is equivocal. Of the following, which study best confirms a diagnosis of acute decompensated heart failure?
A. Brain natriuretic peptide
B. Point-of-care cardiac ultrasound
C. Point-of-care lung ultrasound
D. Point-of-care inferior vena cava ultrasound
Answer
C. Point-of-care lung ultrasound
Multiple studies, including three systematic reviews, have shown that point-of-care lung ultrasound has high sensitivity and specificity to evaluate for B lines as a marker for cardiogenic pulmonary edema. Point-of-care ultrasound of ejection fraction and inferior vena cava have not been evaluated by systematic review although one randomized, controlled trial showed that an EF less than 45% had 74% specificity and 77% sensitivity and IVC collapsibility index less than 20% had an 86% specificity and 52% sensitivity for detection of acute decompensated heart failure. This same study showed that the combination of cardiac, lung, and IVC point-of-care ultrasound had 100% specificity for diagnosing acute decompensated heart failure. In the future, health care providers could rely on this multiorgan evaluation with point-of-care ultrasound to confirm a diagnosis of acute decompensated heart failure in a dyspneic patient.
Advancing bronchoscopy: Reaching the unreachable
As of 2019, lung cancer remained the leading cause of cancer death in the United States. In March 2021, the USPSTF updated the guidelines for lung cancer screening, increasing the number of eligible patients in order to identify malignancies in the early stages when more treatment options exist. With the growth of lung cancer screening, increasingly smaller pulmonary nodules are being identified in more peripheral locations previously thought to be unreachable with bronchoscopy. While bronchoscopy has been utilized for over a century for therapeutic interventions, the development of the fiberoptic bronchoscope in 1967 ushered in an era of evolving diagnostic functions. From the initial endobronchial and transbronchial biopsy techniques, to the introduction of endobronchial ultrasound, and now the latest navigational and robotic modalities, these advances have opened a new realm of interventions available in our diagnostic approach to lung cancer.
Bronchoscopy has become essential in the diagnosis of thoracic malignancies, providing both diagnostic and staging information in one procedural setting. By first assessing the mediastinal and hilar lymph nodes with endobronchial ultrasound and transbronchial needle aspiration, involved lymph nodes can give both diagnosis and staging information required to guide treatment. This is particularly important in the case of non-small cell lung cancer, which utilizes the TNM staging system. Through the use of convex probe endobronchial ultrasound (CP-EBUS), combined with rapid on-site evaluation (ROSE) by pathologic condition, we can more accurately target the individual lymph nodes for biopsy without the need for any additional procedures that are often more complex and invasive, such as mediastinoscopy. It is important to note the role of CP-EBUS extends beyond the lymph node assessment and can also be utilized for the evaluation of other mediastinal lesions, such as central parenchymal masses. These would otherwise be difficult to access due to the lack of a clear airway to the lesion (Argento and Puchalski. Respir Med. 2016;116:55-8).
While EBUS has improved the sampling of lymph nodes, advanced imaging technologies and subsequent increases in lung cancer screening have increased the number of lung malignancies identified in earlier stages before extension to the lymph nodes occurs. This scenario requires a direct biopsy of the primary nodule or lung mass. While CP-EBUS can be utilized for some central parenchymal lesions, peripheral nodules pose a greater challenge to the bronchoscopist as they cannot be directly visualized with the conventional bronchoscope. These lesions are amenable to traditional sampling techniques such as bronchial brushings and washings in addition to transbronchial needle aspiration and transbronchial biopsy. However, the yield for peripheral lesions is less than that for central tumors and depends on lesion size, distance from hilum, spatial positioning from bronchus, and operator experience. To help localize peripheral lesions, a separate form of endobronchial ultrasound is available that can be used in combination with fluoroscopy to target a lesion. Radial probe endobronchial ultrasound (RP-EBUS) utilizes a rotating ultrasound transducer that can be advanced either through the working channel of the bronchoscope or through a guide sheath to extend to airways beyond what the conventional bronchoscope can reach. This assists the bronchoscopist with locating the correct airway and, therefore, increases the yield of sampling techniques. The use of RP-EBUS has reported diagnostic yields of almost 85% if the ultrasound is located within the lesion, but less than 50% if adjacent to the lesion (Chen et al. Ann Am Thorac Soc. 2014;11[4]:578-82). While this improves the yield beyond that achieved with conventional bronchoscopy alone, it continues to challenge the bronchoscopist to locate an accessible airway from a series of branching bronchi that are beyond the level of direct visualization.
Due to the historical difficulty in accurately reaching peripheral lesions, alternative technologies for sampling these lesions, such as image-guided biopsies or surgical resection, were employed. While CT scan-guided biopsies traditionally have high diagnostic yields, they also carry a higher rate of complications, including pneumothorax and bleeding. This has led to a significant increase over the past 2 decades in new bronchoscopic technologies targeting safer and more accurate sampling of increasingly smaller, peripheral lesions.
Traditionally, any new technologies created were intended to be used alongside flexible fiberoptic bronchoscopy. The more recently introduced technologies, however, aim to provide a safer, more accurate procedure through virtual bronchoscopy. By obtaining CT scan images prior to the procedure, a 3D visualization is constructed of the tracheobronchial tree, allowing for directed guidance of endobronchial accessories to more distal airways. Where the bronchoscopist was previously limited in navigating the bronchial tree to the subsegmental bronchi, virtual bronchoscopy can depict the airways up to the 7th order subdivision. This is a significant improvement in airway visualization – however, only when partnered with guidance technologies can the model be accurately navigated.
One modality that is often coupled with virtual bronchoscopy to accurately reach peripheral lesions is electromagnetic navigation bronchoscopy (ENB). Multiple ENB software systems have been created and continue to be highly utilized by bronchoscopists to target peripheral lesions, as it has often been likened to a GPS for the lungs. With the addition of specific hardware components, a magnetic field is created around the patient where the sensor position can be elicited to within 1-mm accuracy. When overlaid with the CT scan images, the bronchoscopist can have real-time positioning of the probe in all three planes and guide the necessary sampling tools to the lesion of interest. The reported yields for ENB vary but have been shown to increase in the presence of specific image findings such as a positive bronchus sign – an air-filled bronchus leading into the lesion. The presence of this finding can increase the yield up to almost 75% from just under 50% in the absence of a positive bronchus sign. (Ali et al. Ann Am Thorac Soc. 2018;15[8]:978-87). However, regardless of this finding, the overall diagnostic yields for ENB continue to fall below that seen with other image-guided biopsy techniques. The procedural complications, however, are significantly less and, therefore, many physicians continue to advocate for ENB as the initial procedure in attempt to decrease risk for the patient.
The newest technology to be introduced to target peripheral lung lesions and to improve upon the shortcomings of other techniques is robotic-assisted bronchoscopy. While surgical specialties have seen success with robotic techniques over many years, the first robotic bronchoscopy system was not introduced until 2018. At present, there are two systems available: the Monarch® system by Auris Health and the Ion Endoluminal® System by Intuitive Surgical. These systems allow for increased bronchoscope stability, improved visualization, adjustable angulation of biopsy tools, and an improved ability to make even subtle turns in the airways. Early studies on both systems were cadaver based, but an increasing number of patient trials are now being reported or actively enrolling. Both systems have shown high rates of lesion localization, greater than 85%, with varying diagnostic yields from 69-79%. Some cadaver studies that utilized artificial tumors reported higher diagnostic yields – over 90% – but this was not seen in initial patient-based studies. (Agrawal et al. J Thorac Dis. 2020;12[6]:3279-86) More data related to the robotic-assisted bronchoscopy systems can be expected in the future as more experience is gained, but initial results are promising in the system’s ability to diagnose early lung cancers safely and accurately.
With increasing technologies and equipment available, bronchoscopy has quickly become an essential step in the diagnosis of lung cancer. While other techniques exist beyond those described here, these are some of the more widely used options currently available. It is not possible at this time to define one technology as the best tool for the diagnosis of lung cancer, as patient factors will always have to be taken into consideration to ensure safety and accuracy. However, with constantly changing technologies, the bronchoscopist now has a variety of tools available to help target previously “unreachable” lesions as we aim to decrease the historically high mortality rates of lung cancer.
Dr. Jewani and Dr. Johnson are from Loyola University Medical Center, Department of Pulmonary and Critical Care Medicine, Maywood, Illinois.
1. Agrawal, Abhinav et al. “Robotic bronchoscopy for pulmonary lesions: a review of existing technologies and clinical data.” Journal of thoracic disease vol. 12,6 (2020): 3279-3286. doi:10.21037/jtd.2020.03.35
2. Ali MS, Sethi J, Taneja A, Musani A, Maldonado F. Computed Tomography Bronchus Sign and the Diagnostic Yield of Guided Bronchoscopy for Peripheral Pulmonary Lesions. A Systematic Review and Meta-Analysis. Ann Am Thorac Soc. 2018 Aug;15(8):978-987. doi: 10.1513/AnnalsATS.201711-856OC. PMID: 29877715.
3. Argento AC, Puchalski J. Convex probe EBUS for centrally located parenchymal lesions without a bronchus sign. Respir Med. 2016 Jul;116:55-8. doi: 10.1016/j.rmed.2016.04.012. Epub 2016 Apr 29. PMID: 27296821.
4. Chen A, Chenna P, Loiselle A, Massoni J, Mayse M, Misselhorn D. Radial probe endobronchial ultrasound for peripheral pulmonary lesions. A 5-year institutional experience. Ann Am Thorac Soc. 2014 May;11(4):578-82. doi: 10.1513/AnnalsATS.201311-384OC. PMID: 24635641.
As of 2019, lung cancer remained the leading cause of cancer death in the United States. In March 2021, the USPSTF updated the guidelines for lung cancer screening, increasing the number of eligible patients in order to identify malignancies in the early stages when more treatment options exist. With the growth of lung cancer screening, increasingly smaller pulmonary nodules are being identified in more peripheral locations previously thought to be unreachable with bronchoscopy. While bronchoscopy has been utilized for over a century for therapeutic interventions, the development of the fiberoptic bronchoscope in 1967 ushered in an era of evolving diagnostic functions. From the initial endobronchial and transbronchial biopsy techniques, to the introduction of endobronchial ultrasound, and now the latest navigational and robotic modalities, these advances have opened a new realm of interventions available in our diagnostic approach to lung cancer.
Bronchoscopy has become essential in the diagnosis of thoracic malignancies, providing both diagnostic and staging information in one procedural setting. By first assessing the mediastinal and hilar lymph nodes with endobronchial ultrasound and transbronchial needle aspiration, involved lymph nodes can give both diagnosis and staging information required to guide treatment. This is particularly important in the case of non-small cell lung cancer, which utilizes the TNM staging system. Through the use of convex probe endobronchial ultrasound (CP-EBUS), combined with rapid on-site evaluation (ROSE) by pathologic condition, we can more accurately target the individual lymph nodes for biopsy without the need for any additional procedures that are often more complex and invasive, such as mediastinoscopy. It is important to note the role of CP-EBUS extends beyond the lymph node assessment and can also be utilized for the evaluation of other mediastinal lesions, such as central parenchymal masses. These would otherwise be difficult to access due to the lack of a clear airway to the lesion (Argento and Puchalski. Respir Med. 2016;116:55-8).
While EBUS has improved the sampling of lymph nodes, advanced imaging technologies and subsequent increases in lung cancer screening have increased the number of lung malignancies identified in earlier stages before extension to the lymph nodes occurs. This scenario requires a direct biopsy of the primary nodule or lung mass. While CP-EBUS can be utilized for some central parenchymal lesions, peripheral nodules pose a greater challenge to the bronchoscopist as they cannot be directly visualized with the conventional bronchoscope. These lesions are amenable to traditional sampling techniques such as bronchial brushings and washings in addition to transbronchial needle aspiration and transbronchial biopsy. However, the yield for peripheral lesions is less than that for central tumors and depends on lesion size, distance from hilum, spatial positioning from bronchus, and operator experience. To help localize peripheral lesions, a separate form of endobronchial ultrasound is available that can be used in combination with fluoroscopy to target a lesion. Radial probe endobronchial ultrasound (RP-EBUS) utilizes a rotating ultrasound transducer that can be advanced either through the working channel of the bronchoscope or through a guide sheath to extend to airways beyond what the conventional bronchoscope can reach. This assists the bronchoscopist with locating the correct airway and, therefore, increases the yield of sampling techniques. The use of RP-EBUS has reported diagnostic yields of almost 85% if the ultrasound is located within the lesion, but less than 50% if adjacent to the lesion (Chen et al. Ann Am Thorac Soc. 2014;11[4]:578-82). While this improves the yield beyond that achieved with conventional bronchoscopy alone, it continues to challenge the bronchoscopist to locate an accessible airway from a series of branching bronchi that are beyond the level of direct visualization.
Due to the historical difficulty in accurately reaching peripheral lesions, alternative technologies for sampling these lesions, such as image-guided biopsies or surgical resection, were employed. While CT scan-guided biopsies traditionally have high diagnostic yields, they also carry a higher rate of complications, including pneumothorax and bleeding. This has led to a significant increase over the past 2 decades in new bronchoscopic technologies targeting safer and more accurate sampling of increasingly smaller, peripheral lesions.
Traditionally, any new technologies created were intended to be used alongside flexible fiberoptic bronchoscopy. The more recently introduced technologies, however, aim to provide a safer, more accurate procedure through virtual bronchoscopy. By obtaining CT scan images prior to the procedure, a 3D visualization is constructed of the tracheobronchial tree, allowing for directed guidance of endobronchial accessories to more distal airways. Where the bronchoscopist was previously limited in navigating the bronchial tree to the subsegmental bronchi, virtual bronchoscopy can depict the airways up to the 7th order subdivision. This is a significant improvement in airway visualization – however, only when partnered with guidance technologies can the model be accurately navigated.
One modality that is often coupled with virtual bronchoscopy to accurately reach peripheral lesions is electromagnetic navigation bronchoscopy (ENB). Multiple ENB software systems have been created and continue to be highly utilized by bronchoscopists to target peripheral lesions, as it has often been likened to a GPS for the lungs. With the addition of specific hardware components, a magnetic field is created around the patient where the sensor position can be elicited to within 1-mm accuracy. When overlaid with the CT scan images, the bronchoscopist can have real-time positioning of the probe in all three planes and guide the necessary sampling tools to the lesion of interest. The reported yields for ENB vary but have been shown to increase in the presence of specific image findings such as a positive bronchus sign – an air-filled bronchus leading into the lesion. The presence of this finding can increase the yield up to almost 75% from just under 50% in the absence of a positive bronchus sign. (Ali et al. Ann Am Thorac Soc. 2018;15[8]:978-87). However, regardless of this finding, the overall diagnostic yields for ENB continue to fall below that seen with other image-guided biopsy techniques. The procedural complications, however, are significantly less and, therefore, many physicians continue to advocate for ENB as the initial procedure in attempt to decrease risk for the patient.
The newest technology to be introduced to target peripheral lung lesions and to improve upon the shortcomings of other techniques is robotic-assisted bronchoscopy. While surgical specialties have seen success with robotic techniques over many years, the first robotic bronchoscopy system was not introduced until 2018. At present, there are two systems available: the Monarch® system by Auris Health and the Ion Endoluminal® System by Intuitive Surgical. These systems allow for increased bronchoscope stability, improved visualization, adjustable angulation of biopsy tools, and an improved ability to make even subtle turns in the airways. Early studies on both systems were cadaver based, but an increasing number of patient trials are now being reported or actively enrolling. Both systems have shown high rates of lesion localization, greater than 85%, with varying diagnostic yields from 69-79%. Some cadaver studies that utilized artificial tumors reported higher diagnostic yields – over 90% – but this was not seen in initial patient-based studies. (Agrawal et al. J Thorac Dis. 2020;12[6]:3279-86) More data related to the robotic-assisted bronchoscopy systems can be expected in the future as more experience is gained, but initial results are promising in the system’s ability to diagnose early lung cancers safely and accurately.
With increasing technologies and equipment available, bronchoscopy has quickly become an essential step in the diagnosis of lung cancer. While other techniques exist beyond those described here, these are some of the more widely used options currently available. It is not possible at this time to define one technology as the best tool for the diagnosis of lung cancer, as patient factors will always have to be taken into consideration to ensure safety and accuracy. However, with constantly changing technologies, the bronchoscopist now has a variety of tools available to help target previously “unreachable” lesions as we aim to decrease the historically high mortality rates of lung cancer.
Dr. Jewani and Dr. Johnson are from Loyola University Medical Center, Department of Pulmonary and Critical Care Medicine, Maywood, Illinois.
1. Agrawal, Abhinav et al. “Robotic bronchoscopy for pulmonary lesions: a review of existing technologies and clinical data.” Journal of thoracic disease vol. 12,6 (2020): 3279-3286. doi:10.21037/jtd.2020.03.35
2. Ali MS, Sethi J, Taneja A, Musani A, Maldonado F. Computed Tomography Bronchus Sign and the Diagnostic Yield of Guided Bronchoscopy for Peripheral Pulmonary Lesions. A Systematic Review and Meta-Analysis. Ann Am Thorac Soc. 2018 Aug;15(8):978-987. doi: 10.1513/AnnalsATS.201711-856OC. PMID: 29877715.
3. Argento AC, Puchalski J. Convex probe EBUS for centrally located parenchymal lesions without a bronchus sign. Respir Med. 2016 Jul;116:55-8. doi: 10.1016/j.rmed.2016.04.012. Epub 2016 Apr 29. PMID: 27296821.
4. Chen A, Chenna P, Loiselle A, Massoni J, Mayse M, Misselhorn D. Radial probe endobronchial ultrasound for peripheral pulmonary lesions. A 5-year institutional experience. Ann Am Thorac Soc. 2014 May;11(4):578-82. doi: 10.1513/AnnalsATS.201311-384OC. PMID: 24635641.
As of 2019, lung cancer remained the leading cause of cancer death in the United States. In March 2021, the USPSTF updated the guidelines for lung cancer screening, increasing the number of eligible patients in order to identify malignancies in the early stages when more treatment options exist. With the growth of lung cancer screening, increasingly smaller pulmonary nodules are being identified in more peripheral locations previously thought to be unreachable with bronchoscopy. While bronchoscopy has been utilized for over a century for therapeutic interventions, the development of the fiberoptic bronchoscope in 1967 ushered in an era of evolving diagnostic functions. From the initial endobronchial and transbronchial biopsy techniques, to the introduction of endobronchial ultrasound, and now the latest navigational and robotic modalities, these advances have opened a new realm of interventions available in our diagnostic approach to lung cancer.
Bronchoscopy has become essential in the diagnosis of thoracic malignancies, providing both diagnostic and staging information in one procedural setting. By first assessing the mediastinal and hilar lymph nodes with endobronchial ultrasound and transbronchial needle aspiration, involved lymph nodes can give both diagnosis and staging information required to guide treatment. This is particularly important in the case of non-small cell lung cancer, which utilizes the TNM staging system. Through the use of convex probe endobronchial ultrasound (CP-EBUS), combined with rapid on-site evaluation (ROSE) by pathologic condition, we can more accurately target the individual lymph nodes for biopsy without the need for any additional procedures that are often more complex and invasive, such as mediastinoscopy. It is important to note the role of CP-EBUS extends beyond the lymph node assessment and can also be utilized for the evaluation of other mediastinal lesions, such as central parenchymal masses. These would otherwise be difficult to access due to the lack of a clear airway to the lesion (Argento and Puchalski. Respir Med. 2016;116:55-8).
While EBUS has improved the sampling of lymph nodes, advanced imaging technologies and subsequent increases in lung cancer screening have increased the number of lung malignancies identified in earlier stages before extension to the lymph nodes occurs. This scenario requires a direct biopsy of the primary nodule or lung mass. While CP-EBUS can be utilized for some central parenchymal lesions, peripheral nodules pose a greater challenge to the bronchoscopist as they cannot be directly visualized with the conventional bronchoscope. These lesions are amenable to traditional sampling techniques such as bronchial brushings and washings in addition to transbronchial needle aspiration and transbronchial biopsy. However, the yield for peripheral lesions is less than that for central tumors and depends on lesion size, distance from hilum, spatial positioning from bronchus, and operator experience. To help localize peripheral lesions, a separate form of endobronchial ultrasound is available that can be used in combination with fluoroscopy to target a lesion. Radial probe endobronchial ultrasound (RP-EBUS) utilizes a rotating ultrasound transducer that can be advanced either through the working channel of the bronchoscope or through a guide sheath to extend to airways beyond what the conventional bronchoscope can reach. This assists the bronchoscopist with locating the correct airway and, therefore, increases the yield of sampling techniques. The use of RP-EBUS has reported diagnostic yields of almost 85% if the ultrasound is located within the lesion, but less than 50% if adjacent to the lesion (Chen et al. Ann Am Thorac Soc. 2014;11[4]:578-82). While this improves the yield beyond that achieved with conventional bronchoscopy alone, it continues to challenge the bronchoscopist to locate an accessible airway from a series of branching bronchi that are beyond the level of direct visualization.
Due to the historical difficulty in accurately reaching peripheral lesions, alternative technologies for sampling these lesions, such as image-guided biopsies or surgical resection, were employed. While CT scan-guided biopsies traditionally have high diagnostic yields, they also carry a higher rate of complications, including pneumothorax and bleeding. This has led to a significant increase over the past 2 decades in new bronchoscopic technologies targeting safer and more accurate sampling of increasingly smaller, peripheral lesions.
Traditionally, any new technologies created were intended to be used alongside flexible fiberoptic bronchoscopy. The more recently introduced technologies, however, aim to provide a safer, more accurate procedure through virtual bronchoscopy. By obtaining CT scan images prior to the procedure, a 3D visualization is constructed of the tracheobronchial tree, allowing for directed guidance of endobronchial accessories to more distal airways. Where the bronchoscopist was previously limited in navigating the bronchial tree to the subsegmental bronchi, virtual bronchoscopy can depict the airways up to the 7th order subdivision. This is a significant improvement in airway visualization – however, only when partnered with guidance technologies can the model be accurately navigated.
One modality that is often coupled with virtual bronchoscopy to accurately reach peripheral lesions is electromagnetic navigation bronchoscopy (ENB). Multiple ENB software systems have been created and continue to be highly utilized by bronchoscopists to target peripheral lesions, as it has often been likened to a GPS for the lungs. With the addition of specific hardware components, a magnetic field is created around the patient where the sensor position can be elicited to within 1-mm accuracy. When overlaid with the CT scan images, the bronchoscopist can have real-time positioning of the probe in all three planes and guide the necessary sampling tools to the lesion of interest. The reported yields for ENB vary but have been shown to increase in the presence of specific image findings such as a positive bronchus sign – an air-filled bronchus leading into the lesion. The presence of this finding can increase the yield up to almost 75% from just under 50% in the absence of a positive bronchus sign. (Ali et al. Ann Am Thorac Soc. 2018;15[8]:978-87). However, regardless of this finding, the overall diagnostic yields for ENB continue to fall below that seen with other image-guided biopsy techniques. The procedural complications, however, are significantly less and, therefore, many physicians continue to advocate for ENB as the initial procedure in attempt to decrease risk for the patient.
The newest technology to be introduced to target peripheral lung lesions and to improve upon the shortcomings of other techniques is robotic-assisted bronchoscopy. While surgical specialties have seen success with robotic techniques over many years, the first robotic bronchoscopy system was not introduced until 2018. At present, there are two systems available: the Monarch® system by Auris Health and the Ion Endoluminal® System by Intuitive Surgical. These systems allow for increased bronchoscope stability, improved visualization, adjustable angulation of biopsy tools, and an improved ability to make even subtle turns in the airways. Early studies on both systems were cadaver based, but an increasing number of patient trials are now being reported or actively enrolling. Both systems have shown high rates of lesion localization, greater than 85%, with varying diagnostic yields from 69-79%. Some cadaver studies that utilized artificial tumors reported higher diagnostic yields – over 90% – but this was not seen in initial patient-based studies. (Agrawal et al. J Thorac Dis. 2020;12[6]:3279-86) More data related to the robotic-assisted bronchoscopy systems can be expected in the future as more experience is gained, but initial results are promising in the system’s ability to diagnose early lung cancers safely and accurately.
With increasing technologies and equipment available, bronchoscopy has quickly become an essential step in the diagnosis of lung cancer. While other techniques exist beyond those described here, these are some of the more widely used options currently available. It is not possible at this time to define one technology as the best tool for the diagnosis of lung cancer, as patient factors will always have to be taken into consideration to ensure safety and accuracy. However, with constantly changing technologies, the bronchoscopist now has a variety of tools available to help target previously “unreachable” lesions as we aim to decrease the historically high mortality rates of lung cancer.
Dr. Jewani and Dr. Johnson are from Loyola University Medical Center, Department of Pulmonary and Critical Care Medicine, Maywood, Illinois.
1. Agrawal, Abhinav et al. “Robotic bronchoscopy for pulmonary lesions: a review of existing technologies and clinical data.” Journal of thoracic disease vol. 12,6 (2020): 3279-3286. doi:10.21037/jtd.2020.03.35
2. Ali MS, Sethi J, Taneja A, Musani A, Maldonado F. Computed Tomography Bronchus Sign and the Diagnostic Yield of Guided Bronchoscopy for Peripheral Pulmonary Lesions. A Systematic Review and Meta-Analysis. Ann Am Thorac Soc. 2018 Aug;15(8):978-987. doi: 10.1513/AnnalsATS.201711-856OC. PMID: 29877715.
3. Argento AC, Puchalski J. Convex probe EBUS for centrally located parenchymal lesions without a bronchus sign. Respir Med. 2016 Jul;116:55-8. doi: 10.1016/j.rmed.2016.04.012. Epub 2016 Apr 29. PMID: 27296821.
4. Chen A, Chenna P, Loiselle A, Massoni J, Mayse M, Misselhorn D. Radial probe endobronchial ultrasound for peripheral pulmonary lesions. A 5-year institutional experience. Ann Am Thorac Soc. 2014 May;11(4):578-82. doi: 10.1513/AnnalsATS.201311-384OC. PMID: 24635641.
Exposure to marijuana smoke linked to increased risk of respiratory infections in children
Exposure to secondhand marijuana smoke is more strongly associated with viral respiratory infections in children, compared with children who were exposed to tobacco smoke and those with no smoke exposure, new research shows.
“The findings of this study are interesting and pleasantly raise further questions,” said Kristen Miller, MD, attending physician in the division of pulmonary and sleep medicine at Children’s Hospital of Philadelphia, who was not involved in the study. “Given the robust literature regarding secondhand smoke exposure and the current landscape surrounding marijuana, this is a timely study to evaluate the prevalence of marijuana use and the associated effects of marijuana exposure among children.”
Prior research has linked primary marijuana use with respiratory effects. A 2020 study associated cannabis use with an increased risk of severe bronchitis, lung hyperinflation, and increased central airway resistance. However, according to the Centers for Disease Control and Prevention, there are still a lot of unanswered questions surrounding secondhand marijuana smoke exposure and its effects.
“If kids are exposed to enough secondhand smoke, regardless of what the substance is, they’re going to have some negative health outcomes with it,” study author Adam Johnson, MD, of Wake Forest University, Winston-Salem, N.C., said in an interview.
The study, published in Pediatric Research, looked at rates of reported ED and urgent care visits and specific illnesses – such as otitis media, viral respiratory infections, and asthma exacerbations – among children with marijuana exposure and tobacco exposure.
For the study, Dr. Johnson and colleagues surveyed 1,500 parents and caregivers who went to an academic children’s hospital between Dec. 1, 2015, and July 30, 2017. Researchers found that children exposed to marijuana smoke had higher rates of ED visits at 2.21 within the past 12 months, compared with those exposed to tobacco smoke (2.14 within the past 12 months) and those with no smoke exposure (1.94 within the past 12 months). However, the difference in these visits were not statistically significant.
Researchers saw that children exposed to secondhand marijuana smoke saw a 30% increase in viral respiratory infections, compared with those who were not exposed to tobacco or marijuana smoke, Dr. Johnson said. Caregivers who smoked marijuana reported a rate of 1.31 viral infections in their children within the last year. Meanwhile those who smoked tobacco reported a rate of 1.00 infections within the last 12 months and caregivers who did not smoke reported 1.04 infections within the year.
“It suggests that components in marijuana smoke may depress the body’s immune responses to viral infections in children,” Dr. Miller said in an interview.
When it came to otitis media episodes, children exposed to marijuana had a rate of 0.96 episodes within the past 12 months. Children experiencing secondhand tobacco smoke had a rate of 0.83 episodes and those with no smoke exposure had 0.75 episodes within the past 12 months. Researchers did not note this difference as statistically significant.
When it came to asthma exacerbations, children exposed to marijuana smoke also had statistically insignificantly higher rates of exacerbations, compared with those exposed to tobacco smoke and those not exposed to smoke.
“I think it was surprising that the survey results found that marijuana seemed to be more strongly associated with the viral respiratory infections than tobacco,” Dr. Johnson said. “We know that secondhand tobacco smoke exposure in kids does lead to things like otitis media or ear infections, asthma attacks, and other processes, including colds. It was interesting that we didn’t find that association [in the new study], but we found that with marijuana.”
Dr. Johnson said the findings are especially concerning with increases in the acceptance and accessibility of marijuana as it becomes legalized in many states.
A 2015 study examined the effect of secondhand marijuana smoke exposure. Researchers found that exposure to secondhand marijuana smoke can increase heart rate, have mild to moderate sedative effects and can produce detectable cannabinoid levels in blood and urine. However, another study published in 2012 found that low to moderate primary marijuana use is less harmful to users’ lungs than tobacco exposure.
Dr. Miller added that little is known about how exposure to marijuana smoke can affect the innate responses to pathogens and there is a need to “study this in more detail” to figure out if secondhand marijuana smoke is a risk factor for either an increase in respiratory virus infections or their severity.
“These questions could have considerable implications for the health of our children and public health measures regarding marijuana use,” she explained. “As documented marijuana use increases, health care providers need to be aware of the effects of marijuana use and exposure.”
Neither Dr. Johnson nor Dr. Miller has any relevant financial disclosures.
Exposure to secondhand marijuana smoke is more strongly associated with viral respiratory infections in children, compared with children who were exposed to tobacco smoke and those with no smoke exposure, new research shows.
“The findings of this study are interesting and pleasantly raise further questions,” said Kristen Miller, MD, attending physician in the division of pulmonary and sleep medicine at Children’s Hospital of Philadelphia, who was not involved in the study. “Given the robust literature regarding secondhand smoke exposure and the current landscape surrounding marijuana, this is a timely study to evaluate the prevalence of marijuana use and the associated effects of marijuana exposure among children.”
Prior research has linked primary marijuana use with respiratory effects. A 2020 study associated cannabis use with an increased risk of severe bronchitis, lung hyperinflation, and increased central airway resistance. However, according to the Centers for Disease Control and Prevention, there are still a lot of unanswered questions surrounding secondhand marijuana smoke exposure and its effects.
“If kids are exposed to enough secondhand smoke, regardless of what the substance is, they’re going to have some negative health outcomes with it,” study author Adam Johnson, MD, of Wake Forest University, Winston-Salem, N.C., said in an interview.
The study, published in Pediatric Research, looked at rates of reported ED and urgent care visits and specific illnesses – such as otitis media, viral respiratory infections, and asthma exacerbations – among children with marijuana exposure and tobacco exposure.
For the study, Dr. Johnson and colleagues surveyed 1,500 parents and caregivers who went to an academic children’s hospital between Dec. 1, 2015, and July 30, 2017. Researchers found that children exposed to marijuana smoke had higher rates of ED visits at 2.21 within the past 12 months, compared with those exposed to tobacco smoke (2.14 within the past 12 months) and those with no smoke exposure (1.94 within the past 12 months). However, the difference in these visits were not statistically significant.
Researchers saw that children exposed to secondhand marijuana smoke saw a 30% increase in viral respiratory infections, compared with those who were not exposed to tobacco or marijuana smoke, Dr. Johnson said. Caregivers who smoked marijuana reported a rate of 1.31 viral infections in their children within the last year. Meanwhile those who smoked tobacco reported a rate of 1.00 infections within the last 12 months and caregivers who did not smoke reported 1.04 infections within the year.
“It suggests that components in marijuana smoke may depress the body’s immune responses to viral infections in children,” Dr. Miller said in an interview.
When it came to otitis media episodes, children exposed to marijuana had a rate of 0.96 episodes within the past 12 months. Children experiencing secondhand tobacco smoke had a rate of 0.83 episodes and those with no smoke exposure had 0.75 episodes within the past 12 months. Researchers did not note this difference as statistically significant.
When it came to asthma exacerbations, children exposed to marijuana smoke also had statistically insignificantly higher rates of exacerbations, compared with those exposed to tobacco smoke and those not exposed to smoke.
“I think it was surprising that the survey results found that marijuana seemed to be more strongly associated with the viral respiratory infections than tobacco,” Dr. Johnson said. “We know that secondhand tobacco smoke exposure in kids does lead to things like otitis media or ear infections, asthma attacks, and other processes, including colds. It was interesting that we didn’t find that association [in the new study], but we found that with marijuana.”
Dr. Johnson said the findings are especially concerning with increases in the acceptance and accessibility of marijuana as it becomes legalized in many states.
A 2015 study examined the effect of secondhand marijuana smoke exposure. Researchers found that exposure to secondhand marijuana smoke can increase heart rate, have mild to moderate sedative effects and can produce detectable cannabinoid levels in blood and urine. However, another study published in 2012 found that low to moderate primary marijuana use is less harmful to users’ lungs than tobacco exposure.
Dr. Miller added that little is known about how exposure to marijuana smoke can affect the innate responses to pathogens and there is a need to “study this in more detail” to figure out if secondhand marijuana smoke is a risk factor for either an increase in respiratory virus infections or their severity.
“These questions could have considerable implications for the health of our children and public health measures regarding marijuana use,” she explained. “As documented marijuana use increases, health care providers need to be aware of the effects of marijuana use and exposure.”
Neither Dr. Johnson nor Dr. Miller has any relevant financial disclosures.
Exposure to secondhand marijuana smoke is more strongly associated with viral respiratory infections in children, compared with children who were exposed to tobacco smoke and those with no smoke exposure, new research shows.
“The findings of this study are interesting and pleasantly raise further questions,” said Kristen Miller, MD, attending physician in the division of pulmonary and sleep medicine at Children’s Hospital of Philadelphia, who was not involved in the study. “Given the robust literature regarding secondhand smoke exposure and the current landscape surrounding marijuana, this is a timely study to evaluate the prevalence of marijuana use and the associated effects of marijuana exposure among children.”
Prior research has linked primary marijuana use with respiratory effects. A 2020 study associated cannabis use with an increased risk of severe bronchitis, lung hyperinflation, and increased central airway resistance. However, according to the Centers for Disease Control and Prevention, there are still a lot of unanswered questions surrounding secondhand marijuana smoke exposure and its effects.
“If kids are exposed to enough secondhand smoke, regardless of what the substance is, they’re going to have some negative health outcomes with it,” study author Adam Johnson, MD, of Wake Forest University, Winston-Salem, N.C., said in an interview.
The study, published in Pediatric Research, looked at rates of reported ED and urgent care visits and specific illnesses – such as otitis media, viral respiratory infections, and asthma exacerbations – among children with marijuana exposure and tobacco exposure.
For the study, Dr. Johnson and colleagues surveyed 1,500 parents and caregivers who went to an academic children’s hospital between Dec. 1, 2015, and July 30, 2017. Researchers found that children exposed to marijuana smoke had higher rates of ED visits at 2.21 within the past 12 months, compared with those exposed to tobacco smoke (2.14 within the past 12 months) and those with no smoke exposure (1.94 within the past 12 months). However, the difference in these visits were not statistically significant.
Researchers saw that children exposed to secondhand marijuana smoke saw a 30% increase in viral respiratory infections, compared with those who were not exposed to tobacco or marijuana smoke, Dr. Johnson said. Caregivers who smoked marijuana reported a rate of 1.31 viral infections in their children within the last year. Meanwhile those who smoked tobacco reported a rate of 1.00 infections within the last 12 months and caregivers who did not smoke reported 1.04 infections within the year.
“It suggests that components in marijuana smoke may depress the body’s immune responses to viral infections in children,” Dr. Miller said in an interview.
When it came to otitis media episodes, children exposed to marijuana had a rate of 0.96 episodes within the past 12 months. Children experiencing secondhand tobacco smoke had a rate of 0.83 episodes and those with no smoke exposure had 0.75 episodes within the past 12 months. Researchers did not note this difference as statistically significant.
When it came to asthma exacerbations, children exposed to marijuana smoke also had statistically insignificantly higher rates of exacerbations, compared with those exposed to tobacco smoke and those not exposed to smoke.
“I think it was surprising that the survey results found that marijuana seemed to be more strongly associated with the viral respiratory infections than tobacco,” Dr. Johnson said. “We know that secondhand tobacco smoke exposure in kids does lead to things like otitis media or ear infections, asthma attacks, and other processes, including colds. It was interesting that we didn’t find that association [in the new study], but we found that with marijuana.”
Dr. Johnson said the findings are especially concerning with increases in the acceptance and accessibility of marijuana as it becomes legalized in many states.
A 2015 study examined the effect of secondhand marijuana smoke exposure. Researchers found that exposure to secondhand marijuana smoke can increase heart rate, have mild to moderate sedative effects and can produce detectable cannabinoid levels in blood and urine. However, another study published in 2012 found that low to moderate primary marijuana use is less harmful to users’ lungs than tobacco exposure.
Dr. Miller added that little is known about how exposure to marijuana smoke can affect the innate responses to pathogens and there is a need to “study this in more detail” to figure out if secondhand marijuana smoke is a risk factor for either an increase in respiratory virus infections or their severity.
“These questions could have considerable implications for the health of our children and public health measures regarding marijuana use,” she explained. “As documented marijuana use increases, health care providers need to be aware of the effects of marijuana use and exposure.”
Neither Dr. Johnson nor Dr. Miller has any relevant financial disclosures.
FROM PEDIATRIC RESEARCH
Injectable monoclonal antibodies prevent COVID-19 in trial
published online August 4, 2021, in the New England Journal of Medicine.
according to results of a randomized, double-blind, placebo-controlled clinical trialThe cocktail of the monoclonal antibodies casirivimab and imdevimab (REGEN-COV, Regeneron Pharmaceuticals) reduced participants’ relative risk of infection by 72%, compared with placebo within the first week. After the first week, risk reduction increased to 93%.
“Long after you would be exposed by your household, there is an enduring effect that prevents you from community spread,” said David Wohl, MD, professor of medicine in the division of infectious diseases at the University of North Carolina at Chapel Hill, who was a site investigator for the trial but not a study author.
Participants were enrolled within 96 hours after someone in their household tested positive for SARS-CoV-2. Participants were randomly assigned to receive 1,200 mg of REGEN-COV subcutaneously or a placebo. Based on serologic testing, study participants showed no evidence of current or previous SARS-CoV-2 infection. The median age of participants was 42.9, but 45% were male teenagers (ages 12-17).
In the group that received REGEN-COV, 11 out of 753 participants developed symptomatic COVID-19, compared with 59 out of 752 participants who received placebo. The relative risk reduction for the study’s 4-week period was 81.4% (P < .001). Of the participants that did develop a SARS-CoV-2 infection, those that received REGEN-COV were less likely to be symptomatic. Asymptomatic infections developed in 25 participants who received REGEN-COV versus 48 in the placebo group. The relative risk of developing any SARS-CoV-2 infection, symptomatic or asymptomatic, was reduced by 66.4% with REGEN-COV (P < .001).
Among the patients who were symptomatic, symptoms subsided within a median of 1.2 weeks for the group that received REGEN-COV, 2 weeks earlier than the placebo group. These patients also had a shorter duration of a high viral load (>104 copies/mL). Few adverse events were reported in the treatment or placebo groups. Monoclonal antibodies “seem to be incredibly safe,” Dr. Wohl said.
“These monoclonal antibodies have proven they can reduce the viral replication in the nose,” said study author Myron Cohen, MD, an infectious disease specialist and professor of epidemiology at the University of North Carolina.
The Food and Drug Administration first granted REGEN-COV emergency use authorization (EUA) in November 2020 for use in patients with mild or moderate COVID-19 who were also at high risk for progressing to severe COVID-19. At that time, the cocktail of monoclonal antibodies was delivered by a single intravenous infusion.
In January, Regeneron first announced the success of this trial of the subcutaneous injection for exposed household contacts based on early results, and in June of 2021, the FDA expanded the EUA to include a subcutaneous delivery when IV is not feasible. On July 30, the EUA was expanded again to include prophylactic use in exposed patients based on these trial results.
The U.S. government has purchased approximately 1.5 million doses of REGEN-COV from Regeneron and has agreed to make the treatments free of charge to patients.
But despite being free, available, and backed by promising data, monoclonal antibodies as a therapeutic answer to COVID-19 still hasn’t really taken off. “The problem is, it first requires knowledge and awareness,” Dr. Wohl said. “A lot [of people] don’t know this exists. To be honest, vaccination has taken up all the oxygen in the room.”
Dr. Cohen agreed. One reason for the slow uptake may be because the drug supply is owned by the government and not a pharmaceutical company. There hasn’t been a typical marketing push to make physicians and consumers aware. Additionally, “the logistics are daunting,” Dr. Cohen said. The office spaces where many physicians care for patients “often aren’t appropriate for patients who think they have SARS-CoV-2.”
“Right now, there’s not a mechanism” to administer the drug to people who could benefit from it, Dr. Wohl said. Eligible patients are either immunocompromised and unlikely to mount a sufficient immune response with vaccination, or not fully vaccinated. They should have been exposed to an infected individual or have a high likelihood of exposure due to where they live, such as in a prison or nursing home. Local doctors are unlikely to be the primary administrators of the drug, Dr. Wohl added. “How do we operationalize this for people who fit the criteria?”
There’s also an issue of timing. REGEN-COV is most effective when given early, Dr. Cohen said. “[Monoclonal antibodies] really only work well in the replication phase.” Many patients who would be eligible delay care until they’ve had symptoms for several days, when REGEN-COV would no longer have the desired effect.
Eventually, Dr. Wohl suspects demand will increase when people realize REGEN-COV can help those with COVID-19 and those who have been exposed. But before then, “we do have to think about how to integrate this into a workflow people can access without being confused.”
The trial was done before there was widespread vaccination, so it’s unclear what the results mean for people who have been vaccinated. Dr. Cohen and Dr. Wohl said there are ongoing conversations about whether monoclonal antibodies could be complementary to vaccination and if there’s potential for continued monthly use of these therapies.
Cohen and Wohl reported no relevant financial relationships. The trial was supported by Regeneron Pharmaceuticals, F. Hoffmann–La Roche, the National Institute of Allergy and Infectious Diseases, NIH, and the COVID-19 Prevention Network.
A version of this article first appeared on Medscape.com.
published online August 4, 2021, in the New England Journal of Medicine.
according to results of a randomized, double-blind, placebo-controlled clinical trialThe cocktail of the monoclonal antibodies casirivimab and imdevimab (REGEN-COV, Regeneron Pharmaceuticals) reduced participants’ relative risk of infection by 72%, compared with placebo within the first week. After the first week, risk reduction increased to 93%.
“Long after you would be exposed by your household, there is an enduring effect that prevents you from community spread,” said David Wohl, MD, professor of medicine in the division of infectious diseases at the University of North Carolina at Chapel Hill, who was a site investigator for the trial but not a study author.
Participants were enrolled within 96 hours after someone in their household tested positive for SARS-CoV-2. Participants were randomly assigned to receive 1,200 mg of REGEN-COV subcutaneously or a placebo. Based on serologic testing, study participants showed no evidence of current or previous SARS-CoV-2 infection. The median age of participants was 42.9, but 45% were male teenagers (ages 12-17).
In the group that received REGEN-COV, 11 out of 753 participants developed symptomatic COVID-19, compared with 59 out of 752 participants who received placebo. The relative risk reduction for the study’s 4-week period was 81.4% (P < .001). Of the participants that did develop a SARS-CoV-2 infection, those that received REGEN-COV were less likely to be symptomatic. Asymptomatic infections developed in 25 participants who received REGEN-COV versus 48 in the placebo group. The relative risk of developing any SARS-CoV-2 infection, symptomatic or asymptomatic, was reduced by 66.4% with REGEN-COV (P < .001).
Among the patients who were symptomatic, symptoms subsided within a median of 1.2 weeks for the group that received REGEN-COV, 2 weeks earlier than the placebo group. These patients also had a shorter duration of a high viral load (>104 copies/mL). Few adverse events were reported in the treatment or placebo groups. Monoclonal antibodies “seem to be incredibly safe,” Dr. Wohl said.
“These monoclonal antibodies have proven they can reduce the viral replication in the nose,” said study author Myron Cohen, MD, an infectious disease specialist and professor of epidemiology at the University of North Carolina.
The Food and Drug Administration first granted REGEN-COV emergency use authorization (EUA) in November 2020 for use in patients with mild or moderate COVID-19 who were also at high risk for progressing to severe COVID-19. At that time, the cocktail of monoclonal antibodies was delivered by a single intravenous infusion.
In January, Regeneron first announced the success of this trial of the subcutaneous injection for exposed household contacts based on early results, and in June of 2021, the FDA expanded the EUA to include a subcutaneous delivery when IV is not feasible. On July 30, the EUA was expanded again to include prophylactic use in exposed patients based on these trial results.
The U.S. government has purchased approximately 1.5 million doses of REGEN-COV from Regeneron and has agreed to make the treatments free of charge to patients.
But despite being free, available, and backed by promising data, monoclonal antibodies as a therapeutic answer to COVID-19 still hasn’t really taken off. “The problem is, it first requires knowledge and awareness,” Dr. Wohl said. “A lot [of people] don’t know this exists. To be honest, vaccination has taken up all the oxygen in the room.”
Dr. Cohen agreed. One reason for the slow uptake may be because the drug supply is owned by the government and not a pharmaceutical company. There hasn’t been a typical marketing push to make physicians and consumers aware. Additionally, “the logistics are daunting,” Dr. Cohen said. The office spaces where many physicians care for patients “often aren’t appropriate for patients who think they have SARS-CoV-2.”
“Right now, there’s not a mechanism” to administer the drug to people who could benefit from it, Dr. Wohl said. Eligible patients are either immunocompromised and unlikely to mount a sufficient immune response with vaccination, or not fully vaccinated. They should have been exposed to an infected individual or have a high likelihood of exposure due to where they live, such as in a prison or nursing home. Local doctors are unlikely to be the primary administrators of the drug, Dr. Wohl added. “How do we operationalize this for people who fit the criteria?”
There’s also an issue of timing. REGEN-COV is most effective when given early, Dr. Cohen said. “[Monoclonal antibodies] really only work well in the replication phase.” Many patients who would be eligible delay care until they’ve had symptoms for several days, when REGEN-COV would no longer have the desired effect.
Eventually, Dr. Wohl suspects demand will increase when people realize REGEN-COV can help those with COVID-19 and those who have been exposed. But before then, “we do have to think about how to integrate this into a workflow people can access without being confused.”
The trial was done before there was widespread vaccination, so it’s unclear what the results mean for people who have been vaccinated. Dr. Cohen and Dr. Wohl said there are ongoing conversations about whether monoclonal antibodies could be complementary to vaccination and if there’s potential for continued monthly use of these therapies.
Cohen and Wohl reported no relevant financial relationships. The trial was supported by Regeneron Pharmaceuticals, F. Hoffmann–La Roche, the National Institute of Allergy and Infectious Diseases, NIH, and the COVID-19 Prevention Network.
A version of this article first appeared on Medscape.com.
published online August 4, 2021, in the New England Journal of Medicine.
according to results of a randomized, double-blind, placebo-controlled clinical trialThe cocktail of the monoclonal antibodies casirivimab and imdevimab (REGEN-COV, Regeneron Pharmaceuticals) reduced participants’ relative risk of infection by 72%, compared with placebo within the first week. After the first week, risk reduction increased to 93%.
“Long after you would be exposed by your household, there is an enduring effect that prevents you from community spread,” said David Wohl, MD, professor of medicine in the division of infectious diseases at the University of North Carolina at Chapel Hill, who was a site investigator for the trial but not a study author.
Participants were enrolled within 96 hours after someone in their household tested positive for SARS-CoV-2. Participants were randomly assigned to receive 1,200 mg of REGEN-COV subcutaneously or a placebo. Based on serologic testing, study participants showed no evidence of current or previous SARS-CoV-2 infection. The median age of participants was 42.9, but 45% were male teenagers (ages 12-17).
In the group that received REGEN-COV, 11 out of 753 participants developed symptomatic COVID-19, compared with 59 out of 752 participants who received placebo. The relative risk reduction for the study’s 4-week period was 81.4% (P < .001). Of the participants that did develop a SARS-CoV-2 infection, those that received REGEN-COV were less likely to be symptomatic. Asymptomatic infections developed in 25 participants who received REGEN-COV versus 48 in the placebo group. The relative risk of developing any SARS-CoV-2 infection, symptomatic or asymptomatic, was reduced by 66.4% with REGEN-COV (P < .001).
Among the patients who were symptomatic, symptoms subsided within a median of 1.2 weeks for the group that received REGEN-COV, 2 weeks earlier than the placebo group. These patients also had a shorter duration of a high viral load (>104 copies/mL). Few adverse events were reported in the treatment or placebo groups. Monoclonal antibodies “seem to be incredibly safe,” Dr. Wohl said.
“These monoclonal antibodies have proven they can reduce the viral replication in the nose,” said study author Myron Cohen, MD, an infectious disease specialist and professor of epidemiology at the University of North Carolina.
The Food and Drug Administration first granted REGEN-COV emergency use authorization (EUA) in November 2020 for use in patients with mild or moderate COVID-19 who were also at high risk for progressing to severe COVID-19. At that time, the cocktail of monoclonal antibodies was delivered by a single intravenous infusion.
In January, Regeneron first announced the success of this trial of the subcutaneous injection for exposed household contacts based on early results, and in June of 2021, the FDA expanded the EUA to include a subcutaneous delivery when IV is not feasible. On July 30, the EUA was expanded again to include prophylactic use in exposed patients based on these trial results.
The U.S. government has purchased approximately 1.5 million doses of REGEN-COV from Regeneron and has agreed to make the treatments free of charge to patients.
But despite being free, available, and backed by promising data, monoclonal antibodies as a therapeutic answer to COVID-19 still hasn’t really taken off. “The problem is, it first requires knowledge and awareness,” Dr. Wohl said. “A lot [of people] don’t know this exists. To be honest, vaccination has taken up all the oxygen in the room.”
Dr. Cohen agreed. One reason for the slow uptake may be because the drug supply is owned by the government and not a pharmaceutical company. There hasn’t been a typical marketing push to make physicians and consumers aware. Additionally, “the logistics are daunting,” Dr. Cohen said. The office spaces where many physicians care for patients “often aren’t appropriate for patients who think they have SARS-CoV-2.”
“Right now, there’s not a mechanism” to administer the drug to people who could benefit from it, Dr. Wohl said. Eligible patients are either immunocompromised and unlikely to mount a sufficient immune response with vaccination, or not fully vaccinated. They should have been exposed to an infected individual or have a high likelihood of exposure due to where they live, such as in a prison or nursing home. Local doctors are unlikely to be the primary administrators of the drug, Dr. Wohl added. “How do we operationalize this for people who fit the criteria?”
There’s also an issue of timing. REGEN-COV is most effective when given early, Dr. Cohen said. “[Monoclonal antibodies] really only work well in the replication phase.” Many patients who would be eligible delay care until they’ve had symptoms for several days, when REGEN-COV would no longer have the desired effect.
Eventually, Dr. Wohl suspects demand will increase when people realize REGEN-COV can help those with COVID-19 and those who have been exposed. But before then, “we do have to think about how to integrate this into a workflow people can access without being confused.”
The trial was done before there was widespread vaccination, so it’s unclear what the results mean for people who have been vaccinated. Dr. Cohen and Dr. Wohl said there are ongoing conversations about whether monoclonal antibodies could be complementary to vaccination and if there’s potential for continued monthly use of these therapies.
Cohen and Wohl reported no relevant financial relationships. The trial was supported by Regeneron Pharmaceuticals, F. Hoffmann–La Roche, the National Institute of Allergy and Infectious Diseases, NIH, and the COVID-19 Prevention Network.
A version of this article first appeared on Medscape.com.
Summer campers spread COVID at home, follow-up finds
In a report published online in The New England Journal of Medicine, researchers found that campers spread COVID to household members after returning home – but transmission was more likely from some than others. Distancing and masking helped reduce the risk.
Victoria T. Chu, MD, MPH, with the Centers for Disease Control and Prevention, Atlanta, and colleagues with the agency and the Georgia Department of Health followed up with 224 camp attendees, aged 7 to 19 years, who had evidence of SARS-CoV-2 infection on laboratory testing.
These index patients – 88% of whom had symptoms – had 526 household contacts, mainly parents and siblings. Of 377 household contacts who underwent testing, 46 (12%) tested positive. Another two cases in household contacts were identified using clinical and epidemiologic criteria.
Family members hospitalized
Of the 41 adult household contacts who were infected, four (about 10%) were hospitalized. Their hospital stays ranged from 5 to 11 days. Of the seven infected household contacts who were younger than 18 years, none were hospitalized.
The four hospitalized adults were parents and grandparents aged 45 to 80 years, Dr. Chu said. Two of the four had underlying conditions. None of the household contacts died.
In an adjusted analysis, campers who had practiced physical distancing were less likely to transmit the virus at home, compared with those who had not practiced physical distancing (adjusted odds ratio, 0.4). Household members who had had close or direct contact with the index patients were more than 5 times more likely to become infected, compared with family members with minimal or no contact, analyses showed.
“This retrospective study showed that the efficient transmission of SARS-CoV-2 from school-age children and adolescents to household members led to the hospitalization of adults with secondary cases of COVID-19,” the researchers write. “In households in which transmission occurred, half the household contacts were infected.”
The secondary attack rates in this report may be an underestimate because testing was voluntary and participants reported the results themselves, the authors note. It is possible that infected household contacts spread the virus further, but this study did not address that question, Dr. Chu said.
For the study, investigators interviewed all camp attendees and their parents or guardians by phone between July 17, 2020 and Aug. 24, 2020, to collect information about demographic and clinical characteristics, SARS-CoV-2 testing, and preventive measures. The researchers’ analysis excluded households in which illness onset in a household contact occurred before or less than 2 days after a camper became sick.
About a third of the index patients began to have symptoms while still at camp. These campers may have been less infectious by the time they got home, compared with those whose symptoms started after they returned.
Two-thirds of the index patients adopted physical distancing at home, which “probably reduced the transmission of SARS-CoV-2 in the household,” Dr. Chu and colleagues wrote.
“Children who have had a known COVID-19 exposure should quarantine and obtain testing if they develop symptoms within the 14 days of returning home,” Dr. Chu advised. “If a child develops COVID-19, the child should be cared for and monitored using the proper combination of physical distancing, isolation when feasible, and mask use to prevent household transmission as much as possible. In addition, any person over the age of 12 is now eligible for vaccination in the United States. If eligible, children attending camp and their family members should get vaccinated to protect themselves and others, as vaccinations are our most effective public health prevention strategy.”
Mitigation can help
Another report regarding four overnight camps in Maine – in which three campers tested positive after they arrived last summer – shows that “aggressive mitigation strategies can be effective” in limiting transmission of the virus, William T. Basco Jr., MD, writes in a commentary for this news organization.
This summer, a range of factors, including vaccination rates at the camp, may influence transmission dynamics, Dr. Chu said in an interview. In July, the Associated Press reported outbreaks tied to summer camps in several states.
“Transmission dynamics will probably vary from summer camp to summer camp depending on many factors, such as vaccination rates of camp attendees, the mitigation measures in place, and the number of individual introductions during camp,” Dr. Chu said. “We would expect that a camp with a low vaccination rate among attendees and no enforcement of mitigation measures” still may experience a large outbreak.
“On the other hand, a large proportion of vaccinated individuals and appropriate implementation of multiple mitigation measures, such as wearing masks, may be quite effective at keeping their transmission rates low,” Dr. Chu added. “For camps with younger children who are not currently eligible for vaccination, implementing layered prevention strategies (e.g., mask use, physical distancing, and encouraging outdoor activities when feasible) is important to prevent transmission.”
Although COVID-19 transmission from children to adults, potentially leading to hospitalization, is not a new phenomenon, “data on the extent of transmission driven by children and adolescents in different settings are still quite sparse,” Dr. Chu said. “A better understanding of their impact on household and community transmission to help guide public health recommendations is particularly important, as most children are still not eligible for vaccination, and in-person schools will be reopening this fall.”
A version of this article first appeared on Medscape.com.
In a report published online in The New England Journal of Medicine, researchers found that campers spread COVID to household members after returning home – but transmission was more likely from some than others. Distancing and masking helped reduce the risk.
Victoria T. Chu, MD, MPH, with the Centers for Disease Control and Prevention, Atlanta, and colleagues with the agency and the Georgia Department of Health followed up with 224 camp attendees, aged 7 to 19 years, who had evidence of SARS-CoV-2 infection on laboratory testing.
These index patients – 88% of whom had symptoms – had 526 household contacts, mainly parents and siblings. Of 377 household contacts who underwent testing, 46 (12%) tested positive. Another two cases in household contacts were identified using clinical and epidemiologic criteria.
Family members hospitalized
Of the 41 adult household contacts who were infected, four (about 10%) were hospitalized. Their hospital stays ranged from 5 to 11 days. Of the seven infected household contacts who were younger than 18 years, none were hospitalized.
The four hospitalized adults were parents and grandparents aged 45 to 80 years, Dr. Chu said. Two of the four had underlying conditions. None of the household contacts died.
In an adjusted analysis, campers who had practiced physical distancing were less likely to transmit the virus at home, compared with those who had not practiced physical distancing (adjusted odds ratio, 0.4). Household members who had had close or direct contact with the index patients were more than 5 times more likely to become infected, compared with family members with minimal or no contact, analyses showed.
“This retrospective study showed that the efficient transmission of SARS-CoV-2 from school-age children and adolescents to household members led to the hospitalization of adults with secondary cases of COVID-19,” the researchers write. “In households in which transmission occurred, half the household contacts were infected.”
The secondary attack rates in this report may be an underestimate because testing was voluntary and participants reported the results themselves, the authors note. It is possible that infected household contacts spread the virus further, but this study did not address that question, Dr. Chu said.
For the study, investigators interviewed all camp attendees and their parents or guardians by phone between July 17, 2020 and Aug. 24, 2020, to collect information about demographic and clinical characteristics, SARS-CoV-2 testing, and preventive measures. The researchers’ analysis excluded households in which illness onset in a household contact occurred before or less than 2 days after a camper became sick.
About a third of the index patients began to have symptoms while still at camp. These campers may have been less infectious by the time they got home, compared with those whose symptoms started after they returned.
Two-thirds of the index patients adopted physical distancing at home, which “probably reduced the transmission of SARS-CoV-2 in the household,” Dr. Chu and colleagues wrote.
“Children who have had a known COVID-19 exposure should quarantine and obtain testing if they develop symptoms within the 14 days of returning home,” Dr. Chu advised. “If a child develops COVID-19, the child should be cared for and monitored using the proper combination of physical distancing, isolation when feasible, and mask use to prevent household transmission as much as possible. In addition, any person over the age of 12 is now eligible for vaccination in the United States. If eligible, children attending camp and their family members should get vaccinated to protect themselves and others, as vaccinations are our most effective public health prevention strategy.”
Mitigation can help
Another report regarding four overnight camps in Maine – in which three campers tested positive after they arrived last summer – shows that “aggressive mitigation strategies can be effective” in limiting transmission of the virus, William T. Basco Jr., MD, writes in a commentary for this news organization.
This summer, a range of factors, including vaccination rates at the camp, may influence transmission dynamics, Dr. Chu said in an interview. In July, the Associated Press reported outbreaks tied to summer camps in several states.
“Transmission dynamics will probably vary from summer camp to summer camp depending on many factors, such as vaccination rates of camp attendees, the mitigation measures in place, and the number of individual introductions during camp,” Dr. Chu said. “We would expect that a camp with a low vaccination rate among attendees and no enforcement of mitigation measures” still may experience a large outbreak.
“On the other hand, a large proportion of vaccinated individuals and appropriate implementation of multiple mitigation measures, such as wearing masks, may be quite effective at keeping their transmission rates low,” Dr. Chu added. “For camps with younger children who are not currently eligible for vaccination, implementing layered prevention strategies (e.g., mask use, physical distancing, and encouraging outdoor activities when feasible) is important to prevent transmission.”
Although COVID-19 transmission from children to adults, potentially leading to hospitalization, is not a new phenomenon, “data on the extent of transmission driven by children and adolescents in different settings are still quite sparse,” Dr. Chu said. “A better understanding of their impact on household and community transmission to help guide public health recommendations is particularly important, as most children are still not eligible for vaccination, and in-person schools will be reopening this fall.”
A version of this article first appeared on Medscape.com.
In a report published online in The New England Journal of Medicine, researchers found that campers spread COVID to household members after returning home – but transmission was more likely from some than others. Distancing and masking helped reduce the risk.
Victoria T. Chu, MD, MPH, with the Centers for Disease Control and Prevention, Atlanta, and colleagues with the agency and the Georgia Department of Health followed up with 224 camp attendees, aged 7 to 19 years, who had evidence of SARS-CoV-2 infection on laboratory testing.
These index patients – 88% of whom had symptoms – had 526 household contacts, mainly parents and siblings. Of 377 household contacts who underwent testing, 46 (12%) tested positive. Another two cases in household contacts were identified using clinical and epidemiologic criteria.
Family members hospitalized
Of the 41 adult household contacts who were infected, four (about 10%) were hospitalized. Their hospital stays ranged from 5 to 11 days. Of the seven infected household contacts who were younger than 18 years, none were hospitalized.
The four hospitalized adults were parents and grandparents aged 45 to 80 years, Dr. Chu said. Two of the four had underlying conditions. None of the household contacts died.
In an adjusted analysis, campers who had practiced physical distancing were less likely to transmit the virus at home, compared with those who had not practiced physical distancing (adjusted odds ratio, 0.4). Household members who had had close or direct contact with the index patients were more than 5 times more likely to become infected, compared with family members with minimal or no contact, analyses showed.
“This retrospective study showed that the efficient transmission of SARS-CoV-2 from school-age children and adolescents to household members led to the hospitalization of adults with secondary cases of COVID-19,” the researchers write. “In households in which transmission occurred, half the household contacts were infected.”
The secondary attack rates in this report may be an underestimate because testing was voluntary and participants reported the results themselves, the authors note. It is possible that infected household contacts spread the virus further, but this study did not address that question, Dr. Chu said.
For the study, investigators interviewed all camp attendees and their parents or guardians by phone between July 17, 2020 and Aug. 24, 2020, to collect information about demographic and clinical characteristics, SARS-CoV-2 testing, and preventive measures. The researchers’ analysis excluded households in which illness onset in a household contact occurred before or less than 2 days after a camper became sick.
About a third of the index patients began to have symptoms while still at camp. These campers may have been less infectious by the time they got home, compared with those whose symptoms started after they returned.
Two-thirds of the index patients adopted physical distancing at home, which “probably reduced the transmission of SARS-CoV-2 in the household,” Dr. Chu and colleagues wrote.
“Children who have had a known COVID-19 exposure should quarantine and obtain testing if they develop symptoms within the 14 days of returning home,” Dr. Chu advised. “If a child develops COVID-19, the child should be cared for and monitored using the proper combination of physical distancing, isolation when feasible, and mask use to prevent household transmission as much as possible. In addition, any person over the age of 12 is now eligible for vaccination in the United States. If eligible, children attending camp and their family members should get vaccinated to protect themselves and others, as vaccinations are our most effective public health prevention strategy.”
Mitigation can help
Another report regarding four overnight camps in Maine – in which three campers tested positive after they arrived last summer – shows that “aggressive mitigation strategies can be effective” in limiting transmission of the virus, William T. Basco Jr., MD, writes in a commentary for this news organization.
This summer, a range of factors, including vaccination rates at the camp, may influence transmission dynamics, Dr. Chu said in an interview. In July, the Associated Press reported outbreaks tied to summer camps in several states.
“Transmission dynamics will probably vary from summer camp to summer camp depending on many factors, such as vaccination rates of camp attendees, the mitigation measures in place, and the number of individual introductions during camp,” Dr. Chu said. “We would expect that a camp with a low vaccination rate among attendees and no enforcement of mitigation measures” still may experience a large outbreak.
“On the other hand, a large proportion of vaccinated individuals and appropriate implementation of multiple mitigation measures, such as wearing masks, may be quite effective at keeping their transmission rates low,” Dr. Chu added. “For camps with younger children who are not currently eligible for vaccination, implementing layered prevention strategies (e.g., mask use, physical distancing, and encouraging outdoor activities when feasible) is important to prevent transmission.”
Although COVID-19 transmission from children to adults, potentially leading to hospitalization, is not a new phenomenon, “data on the extent of transmission driven by children and adolescents in different settings are still quite sparse,” Dr. Chu said. “A better understanding of their impact on household and community transmission to help guide public health recommendations is particularly important, as most children are still not eligible for vaccination, and in-person schools will be reopening this fall.”
A version of this article first appeared on Medscape.com.
COVID-19: Delta variant is raising the stakes
Empathetic conversations with unvaccinated people desperately needed
Like many colleagues, I have been working to change the minds and behaviors of acquaintances and patients who are opting to forgo a COVID vaccine. The large numbers of these unvaccinated Americans, combined with the surging Delta coronavirus variant, are endangering the health of us all.
When I spoke with the 22-year-old daughter of a family friend about what was holding her back, she told me that she would “never” get vaccinated. I shared my vaccination experience and told her that, except for a sore arm both times for a day, I felt no side effects. Likewise, I said, all of my adult family members are vaccinated, and everyone is fine. She was neither moved nor convinced.
Finally, I asked her whether she attended school (knowing that she was a college graduate), and she said “yes.” So I told her that all 50 states require children attending public schools to be vaccinated for diseases such as diphtheria, tetanus, polio, and the chickenpox – with certain religious, philosophical, and medical exemptions. Her response was simple: “I didn’t know that. Anyway, my parents were in charge.” Suddenly, her thinking shifted. “You’re right,” she said. She got a COVID shot the next day. Success for me.
When I asked another acquaintance whether he’d been vaccinated, he said he’d heard people were getting very sick from the vaccine – and was going to wait. Another gentleman I spoke with said that, at age 45, he was healthy. Besides, he added, he “doesn’t get sick.” When I asked another acquaintance about her vaccination status, her retort was that this was none of my business. So far, I’m batting about .300.
But as a physician, I believe that we – and other health care providers – must continue to encourage the people in our lives to care for themselves and others by getting vaccinated. One concrete step advised by the Centers for Disease Control and Prevention is to help people make an appointment for a shot. Some sites no longer require appointments, and New York City, for example, offers in-home vaccinations to all NYC residents.
Also, NYC Mayor Bill de Blasio announced Aug. 3 the “Key to NYC Pass,” which he called a “first-in-the-nation approach” to vaccination. Under this new policy, vaccine-eligible people aged 12 and older in New York City will need to prove with a vaccination card, an app, or an Excelsior Pass that they have received at least one dose of vaccine before participating in indoor venues such as restaurants, bars, gyms, and movie theaters within the city. Mayor de Blasio said the new initiative, which is still being finalized, will be phased in starting the week of Aug. 16. I see this as a major public health measure that will keep people healthy – and get them vaccinated.
The medical community should support this move by the city of New York and encourage people to follow CDC guidance on wearing face coverings in public settings, especially schools. New research shows that physicians continue to be among the most trusted sources of vaccine-related information.
Another strategy we might use is to point to the longtime practices of surgeons. We could ask: Why do surgeons wear face masks in the operating room? For years, these coverings have been used to protect patients from the nasal and oral bacteria generated by operating room staff. Likewise, we can tell those who remain on the fence that, by wearing face masks, we are protecting others from all variants, but specifically from Delta – which the CDC now says can be transmitted by people who are fully vaccinated.
Why did the CDC lift face mask guidance for fully vaccinated people in indoor spaces in May? It was clear to me and other colleagues back then that this was not a good idea. Despite that guidance, I continued to wear a mask in public places and advised anyone who would listen to do the same.
The development of vaccines in the 20th and 21st centuries has saved millions of lives. The World Health Organization reports that 4 million to 5 million lives a year are saved by immunizations. In addition, research shows that, before the emergence of SARS-CoV-2, vaccinations led to the eradication of smallpox and polio, and a 74% drop in measles-related deaths between 2004 and 2014.
Protecting the most vulnerable
With COVID cases surging, particularly in parts of the South and Midwest, I am concerned about children under age 12 who do not yet qualify for a vaccine. Certainly, unvaccinated parents could spread the virus to their young children, and unvaccinated children could transmit the illness to immediate and extended family. Now that the CDC has said that there is a risk of SARS-CoV-2 breakthrough infection among fully vaccinated people in areas with high community transmission, should we worry about unvaccinated young children with vaccinated parents? I recently spoke with James C. Fagin, MD, a board-certified pediatrician and immunologist, to get his views on this issue.
Dr. Fagin, who is retired, said he is in complete agreement with the Food and Drug Administration when it comes to approving medications for children. However, given the seriousness of the pandemic and the need to get our children back to in-person learning, he would like to see the approval process safely expedited. Large numbers of unvaccinated people increase the pool for the Delta variant and could increase the likelihood of a new variant that is more resistant to the vaccines, said Dr. Fagin, former chief of academic pediatrics at North Shore University Hospital and a former faculty member in the allergy/immunology division of Cohen Children’s Medical Center, both in New York.
Meanwhile, I agree with the American Academy of Pediatrics’ recommendations that children, teachers, and school staff and other adults in school settings should wear masks regardless of vaccination status. Kids adjust well to masks – as my grandchildren and their friends have.
The bottom line is that we need to get as many people as possible vaccinated as soon as possible, and while doing so, we must continue to wear face coverings in public spaces. As clinicians, we have a special responsibility to do all that we can to change minds – and behaviors.
Dr. London is a practicing psychiatrist who has been a newspaper columnist for 35 years, specializing in and writing about short-term therapy, including cognitive-behavioral therapy and guided imagery. He is author of “Find Freedom Fast” (New York: Kettlehole Publishing, 2019). He has no conflicts of interest.
Empathetic conversations with unvaccinated people desperately needed
Empathetic conversations with unvaccinated people desperately needed
Like many colleagues, I have been working to change the minds and behaviors of acquaintances and patients who are opting to forgo a COVID vaccine. The large numbers of these unvaccinated Americans, combined with the surging Delta coronavirus variant, are endangering the health of us all.
When I spoke with the 22-year-old daughter of a family friend about what was holding her back, she told me that she would “never” get vaccinated. I shared my vaccination experience and told her that, except for a sore arm both times for a day, I felt no side effects. Likewise, I said, all of my adult family members are vaccinated, and everyone is fine. She was neither moved nor convinced.
Finally, I asked her whether she attended school (knowing that she was a college graduate), and she said “yes.” So I told her that all 50 states require children attending public schools to be vaccinated for diseases such as diphtheria, tetanus, polio, and the chickenpox – with certain religious, philosophical, and medical exemptions. Her response was simple: “I didn’t know that. Anyway, my parents were in charge.” Suddenly, her thinking shifted. “You’re right,” she said. She got a COVID shot the next day. Success for me.
When I asked another acquaintance whether he’d been vaccinated, he said he’d heard people were getting very sick from the vaccine – and was going to wait. Another gentleman I spoke with said that, at age 45, he was healthy. Besides, he added, he “doesn’t get sick.” When I asked another acquaintance about her vaccination status, her retort was that this was none of my business. So far, I’m batting about .300.
But as a physician, I believe that we – and other health care providers – must continue to encourage the people in our lives to care for themselves and others by getting vaccinated. One concrete step advised by the Centers for Disease Control and Prevention is to help people make an appointment for a shot. Some sites no longer require appointments, and New York City, for example, offers in-home vaccinations to all NYC residents.
Also, NYC Mayor Bill de Blasio announced Aug. 3 the “Key to NYC Pass,” which he called a “first-in-the-nation approach” to vaccination. Under this new policy, vaccine-eligible people aged 12 and older in New York City will need to prove with a vaccination card, an app, or an Excelsior Pass that they have received at least one dose of vaccine before participating in indoor venues such as restaurants, bars, gyms, and movie theaters within the city. Mayor de Blasio said the new initiative, which is still being finalized, will be phased in starting the week of Aug. 16. I see this as a major public health measure that will keep people healthy – and get them vaccinated.
The medical community should support this move by the city of New York and encourage people to follow CDC guidance on wearing face coverings in public settings, especially schools. New research shows that physicians continue to be among the most trusted sources of vaccine-related information.
Another strategy we might use is to point to the longtime practices of surgeons. We could ask: Why do surgeons wear face masks in the operating room? For years, these coverings have been used to protect patients from the nasal and oral bacteria generated by operating room staff. Likewise, we can tell those who remain on the fence that, by wearing face masks, we are protecting others from all variants, but specifically from Delta – which the CDC now says can be transmitted by people who are fully vaccinated.
Why did the CDC lift face mask guidance for fully vaccinated people in indoor spaces in May? It was clear to me and other colleagues back then that this was not a good idea. Despite that guidance, I continued to wear a mask in public places and advised anyone who would listen to do the same.
The development of vaccines in the 20th and 21st centuries has saved millions of lives. The World Health Organization reports that 4 million to 5 million lives a year are saved by immunizations. In addition, research shows that, before the emergence of SARS-CoV-2, vaccinations led to the eradication of smallpox and polio, and a 74% drop in measles-related deaths between 2004 and 2014.
Protecting the most vulnerable
With COVID cases surging, particularly in parts of the South and Midwest, I am concerned about children under age 12 who do not yet qualify for a vaccine. Certainly, unvaccinated parents could spread the virus to their young children, and unvaccinated children could transmit the illness to immediate and extended family. Now that the CDC has said that there is a risk of SARS-CoV-2 breakthrough infection among fully vaccinated people in areas with high community transmission, should we worry about unvaccinated young children with vaccinated parents? I recently spoke with James C. Fagin, MD, a board-certified pediatrician and immunologist, to get his views on this issue.
Dr. Fagin, who is retired, said he is in complete agreement with the Food and Drug Administration when it comes to approving medications for children. However, given the seriousness of the pandemic and the need to get our children back to in-person learning, he would like to see the approval process safely expedited. Large numbers of unvaccinated people increase the pool for the Delta variant and could increase the likelihood of a new variant that is more resistant to the vaccines, said Dr. Fagin, former chief of academic pediatrics at North Shore University Hospital and a former faculty member in the allergy/immunology division of Cohen Children’s Medical Center, both in New York.
Meanwhile, I agree with the American Academy of Pediatrics’ recommendations that children, teachers, and school staff and other adults in school settings should wear masks regardless of vaccination status. Kids adjust well to masks – as my grandchildren and their friends have.
The bottom line is that we need to get as many people as possible vaccinated as soon as possible, and while doing so, we must continue to wear face coverings in public spaces. As clinicians, we have a special responsibility to do all that we can to change minds – and behaviors.
Dr. London is a practicing psychiatrist who has been a newspaper columnist for 35 years, specializing in and writing about short-term therapy, including cognitive-behavioral therapy and guided imagery. He is author of “Find Freedom Fast” (New York: Kettlehole Publishing, 2019). He has no conflicts of interest.
Like many colleagues, I have been working to change the minds and behaviors of acquaintances and patients who are opting to forgo a COVID vaccine. The large numbers of these unvaccinated Americans, combined with the surging Delta coronavirus variant, are endangering the health of us all.
When I spoke with the 22-year-old daughter of a family friend about what was holding her back, she told me that she would “never” get vaccinated. I shared my vaccination experience and told her that, except for a sore arm both times for a day, I felt no side effects. Likewise, I said, all of my adult family members are vaccinated, and everyone is fine. She was neither moved nor convinced.
Finally, I asked her whether she attended school (knowing that she was a college graduate), and she said “yes.” So I told her that all 50 states require children attending public schools to be vaccinated for diseases such as diphtheria, tetanus, polio, and the chickenpox – with certain religious, philosophical, and medical exemptions. Her response was simple: “I didn’t know that. Anyway, my parents were in charge.” Suddenly, her thinking shifted. “You’re right,” she said. She got a COVID shot the next day. Success for me.
When I asked another acquaintance whether he’d been vaccinated, he said he’d heard people were getting very sick from the vaccine – and was going to wait. Another gentleman I spoke with said that, at age 45, he was healthy. Besides, he added, he “doesn’t get sick.” When I asked another acquaintance about her vaccination status, her retort was that this was none of my business. So far, I’m batting about .300.
But as a physician, I believe that we – and other health care providers – must continue to encourage the people in our lives to care for themselves and others by getting vaccinated. One concrete step advised by the Centers for Disease Control and Prevention is to help people make an appointment for a shot. Some sites no longer require appointments, and New York City, for example, offers in-home vaccinations to all NYC residents.
Also, NYC Mayor Bill de Blasio announced Aug. 3 the “Key to NYC Pass,” which he called a “first-in-the-nation approach” to vaccination. Under this new policy, vaccine-eligible people aged 12 and older in New York City will need to prove with a vaccination card, an app, or an Excelsior Pass that they have received at least one dose of vaccine before participating in indoor venues such as restaurants, bars, gyms, and movie theaters within the city. Mayor de Blasio said the new initiative, which is still being finalized, will be phased in starting the week of Aug. 16. I see this as a major public health measure that will keep people healthy – and get them vaccinated.
The medical community should support this move by the city of New York and encourage people to follow CDC guidance on wearing face coverings in public settings, especially schools. New research shows that physicians continue to be among the most trusted sources of vaccine-related information.
Another strategy we might use is to point to the longtime practices of surgeons. We could ask: Why do surgeons wear face masks in the operating room? For years, these coverings have been used to protect patients from the nasal and oral bacteria generated by operating room staff. Likewise, we can tell those who remain on the fence that, by wearing face masks, we are protecting others from all variants, but specifically from Delta – which the CDC now says can be transmitted by people who are fully vaccinated.
Why did the CDC lift face mask guidance for fully vaccinated people in indoor spaces in May? It was clear to me and other colleagues back then that this was not a good idea. Despite that guidance, I continued to wear a mask in public places and advised anyone who would listen to do the same.
The development of vaccines in the 20th and 21st centuries has saved millions of lives. The World Health Organization reports that 4 million to 5 million lives a year are saved by immunizations. In addition, research shows that, before the emergence of SARS-CoV-2, vaccinations led to the eradication of smallpox and polio, and a 74% drop in measles-related deaths between 2004 and 2014.
Protecting the most vulnerable
With COVID cases surging, particularly in parts of the South and Midwest, I am concerned about children under age 12 who do not yet qualify for a vaccine. Certainly, unvaccinated parents could spread the virus to their young children, and unvaccinated children could transmit the illness to immediate and extended family. Now that the CDC has said that there is a risk of SARS-CoV-2 breakthrough infection among fully vaccinated people in areas with high community transmission, should we worry about unvaccinated young children with vaccinated parents? I recently spoke with James C. Fagin, MD, a board-certified pediatrician and immunologist, to get his views on this issue.
Dr. Fagin, who is retired, said he is in complete agreement with the Food and Drug Administration when it comes to approving medications for children. However, given the seriousness of the pandemic and the need to get our children back to in-person learning, he would like to see the approval process safely expedited. Large numbers of unvaccinated people increase the pool for the Delta variant and could increase the likelihood of a new variant that is more resistant to the vaccines, said Dr. Fagin, former chief of academic pediatrics at North Shore University Hospital and a former faculty member in the allergy/immunology division of Cohen Children’s Medical Center, both in New York.
Meanwhile, I agree with the American Academy of Pediatrics’ recommendations that children, teachers, and school staff and other adults in school settings should wear masks regardless of vaccination status. Kids adjust well to masks – as my grandchildren and their friends have.
The bottom line is that we need to get as many people as possible vaccinated as soon as possible, and while doing so, we must continue to wear face coverings in public spaces. As clinicians, we have a special responsibility to do all that we can to change minds – and behaviors.
Dr. London is a practicing psychiatrist who has been a newspaper columnist for 35 years, specializing in and writing about short-term therapy, including cognitive-behavioral therapy and guided imagery. He is author of “Find Freedom Fast” (New York: Kettlehole Publishing, 2019). He has no conflicts of interest.
Doctors’ offices may be hot spot for transmission of respiratory infections
Prior research has examined the issue of hospital-acquired infections. A 2014 study published in the New England Journal of Medicine, for example, found that 4% of hospitalized patients acquired a health care–associated infection during their stay. Furthermore, the Centers for Disease Control and Prevention estimates that, on any given day, one in 31 hospital patients has at least one health care–associated infection. However, researchers for the new study, published in Health Affairs, said evidence about the risk of acquiring respiratory viral infections in medical office settings is limited.
“Hospital-acquired infections has been a problem for a while,” study author Hannah Neprash, PhD, of the department of health policy and management at the University of Minnesota School of Public Health, Minneapolis, said in an interview. “However, there’s never been a similar study of whether a similar phenomenon happens in physician offices. This is especially relevant now when we’re dealing with respiratory infections.”
Methods and results
For the new study, Dr. Neprash and her colleagues analyzed deidentified billing and scheduling data from 2016-2017 for 105,462,600 outpatient visits that occurred at 6,709 office-based primary care practices. They used the World Health Organization case definition for influenzalike illness “to capture cases in which the physician may suspect this illness even if a specific diagnosis code was not present.” Their control conditions included exposure to urinary tract infections and back pain.
Doctor visits were considered unexposed if they were scheduled to start at least 90 minutes before the first influenzalike illness visit of the day. They were considered exposed if they were scheduled to start at the same time or after the first influenzalike illness visit of the day at that practice.
Researchers quantified whether exposed patients were more likely to return with a similar illness in the next 2 weeks, compared with nonexposed patients seen earlier in the day
They found that 2.7 patients per 1,000 returned within 2 weeks with an influenzalike illness.
Patients were more likely to return with influenzalike illness if their visit occurred after an influenzalike illness visit versus before, the researchers said.
The authors of the paper said their new research highlights the importance of infection control in health care settings, including outpatient offices.
Where did the exposure occur?
Diego Hijano, MD, MSc, pediatric infectious disease specialist at St. Jude’s Children’s Research Hospital, Memphis, Tenn., said he was not surprised by the findings, but noted that it’s hard to say if the exposure to influenzalike illnesses happened in the office or in the community.
“If you start to see individuals with influenza in your office it’s because [there’s influenza] in the community,” Dr. Hijano explained. “So that means that you will have more patients coming in with influenza.”
To reduce the transmission of infections, Dr. Neprash suggested that doctors’ offices follow the CDC guidelines for indoor conduct, which include masking, washing hands, and “taking appropriate infection control measures.”
So potentially masking within offices is a way to minimize transmission between whatever people are there to be seen when it’s contagious, Dr. Neprash said.
“Telehealth really took off in 2020 and it’s unclear what the state of telehealth will be going forward. [These findings] suggest that there’s a patient safety argument for continuing to enable primary care physicians to provide visits either by phone or by video,” he added.
Dr. Hijano thinks it would be helpful for doctors to separate patients with respiratory illnesses from those without respiratory illnesses.
Driver of transmissions
Dr. Neprash suggested that another driver of these transmissions could be doctors not washing their hands, which is a “notorious issue,” and Dr. Hijano agreed with that statement.
“We did know that the hands of physicians and nurses and care providers are the main driver of infections in the health care setting,” Dr. Hijano explained. “I mean, washing your hands properly between encounters is the single best way that any given health care provider can prevent the spread of infections.”
“We have a unique opportunity with COVID-19 to change how these clinics are operating now,” Dr. Hijano said. “Many clinics are actually asking patients to call ahead of time if you have symptoms of a respiratory illness that could be contagious, and those who are not are still mandating the use of mask and physical distance in the waiting areas and limiting the amount of number of patients in any given hour. So I think that those are really big practices that would kind of make an impact in respiratory illness in terms of decreasing transmission in clinics.”
The authors, who had no conflicts of interest said their hope is that their study will help inform policy for reopening outpatient care settings. Dr. Hijano, who was not involved in the study also had no conflicts.
Prior research has examined the issue of hospital-acquired infections. A 2014 study published in the New England Journal of Medicine, for example, found that 4% of hospitalized patients acquired a health care–associated infection during their stay. Furthermore, the Centers for Disease Control and Prevention estimates that, on any given day, one in 31 hospital patients has at least one health care–associated infection. However, researchers for the new study, published in Health Affairs, said evidence about the risk of acquiring respiratory viral infections in medical office settings is limited.
“Hospital-acquired infections has been a problem for a while,” study author Hannah Neprash, PhD, of the department of health policy and management at the University of Minnesota School of Public Health, Minneapolis, said in an interview. “However, there’s never been a similar study of whether a similar phenomenon happens in physician offices. This is especially relevant now when we’re dealing with respiratory infections.”
Methods and results
For the new study, Dr. Neprash and her colleagues analyzed deidentified billing and scheduling data from 2016-2017 for 105,462,600 outpatient visits that occurred at 6,709 office-based primary care practices. They used the World Health Organization case definition for influenzalike illness “to capture cases in which the physician may suspect this illness even if a specific diagnosis code was not present.” Their control conditions included exposure to urinary tract infections and back pain.
Doctor visits were considered unexposed if they were scheduled to start at least 90 minutes before the first influenzalike illness visit of the day. They were considered exposed if they were scheduled to start at the same time or after the first influenzalike illness visit of the day at that practice.
Researchers quantified whether exposed patients were more likely to return with a similar illness in the next 2 weeks, compared with nonexposed patients seen earlier in the day
They found that 2.7 patients per 1,000 returned within 2 weeks with an influenzalike illness.
Patients were more likely to return with influenzalike illness if their visit occurred after an influenzalike illness visit versus before, the researchers said.
The authors of the paper said their new research highlights the importance of infection control in health care settings, including outpatient offices.
Where did the exposure occur?
Diego Hijano, MD, MSc, pediatric infectious disease specialist at St. Jude’s Children’s Research Hospital, Memphis, Tenn., said he was not surprised by the findings, but noted that it’s hard to say if the exposure to influenzalike illnesses happened in the office or in the community.
“If you start to see individuals with influenza in your office it’s because [there’s influenza] in the community,” Dr. Hijano explained. “So that means that you will have more patients coming in with influenza.”
To reduce the transmission of infections, Dr. Neprash suggested that doctors’ offices follow the CDC guidelines for indoor conduct, which include masking, washing hands, and “taking appropriate infection control measures.”
So potentially masking within offices is a way to minimize transmission between whatever people are there to be seen when it’s contagious, Dr. Neprash said.
“Telehealth really took off in 2020 and it’s unclear what the state of telehealth will be going forward. [These findings] suggest that there’s a patient safety argument for continuing to enable primary care physicians to provide visits either by phone or by video,” he added.
Dr. Hijano thinks it would be helpful for doctors to separate patients with respiratory illnesses from those without respiratory illnesses.
Driver of transmissions
Dr. Neprash suggested that another driver of these transmissions could be doctors not washing their hands, which is a “notorious issue,” and Dr. Hijano agreed with that statement.
“We did know that the hands of physicians and nurses and care providers are the main driver of infections in the health care setting,” Dr. Hijano explained. “I mean, washing your hands properly between encounters is the single best way that any given health care provider can prevent the spread of infections.”
“We have a unique opportunity with COVID-19 to change how these clinics are operating now,” Dr. Hijano said. “Many clinics are actually asking patients to call ahead of time if you have symptoms of a respiratory illness that could be contagious, and those who are not are still mandating the use of mask and physical distance in the waiting areas and limiting the amount of number of patients in any given hour. So I think that those are really big practices that would kind of make an impact in respiratory illness in terms of decreasing transmission in clinics.”
The authors, who had no conflicts of interest said their hope is that their study will help inform policy for reopening outpatient care settings. Dr. Hijano, who was not involved in the study also had no conflicts.
Prior research has examined the issue of hospital-acquired infections. A 2014 study published in the New England Journal of Medicine, for example, found that 4% of hospitalized patients acquired a health care–associated infection during their stay. Furthermore, the Centers for Disease Control and Prevention estimates that, on any given day, one in 31 hospital patients has at least one health care–associated infection. However, researchers for the new study, published in Health Affairs, said evidence about the risk of acquiring respiratory viral infections in medical office settings is limited.
“Hospital-acquired infections has been a problem for a while,” study author Hannah Neprash, PhD, of the department of health policy and management at the University of Minnesota School of Public Health, Minneapolis, said in an interview. “However, there’s never been a similar study of whether a similar phenomenon happens in physician offices. This is especially relevant now when we’re dealing with respiratory infections.”
Methods and results
For the new study, Dr. Neprash and her colleagues analyzed deidentified billing and scheduling data from 2016-2017 for 105,462,600 outpatient visits that occurred at 6,709 office-based primary care practices. They used the World Health Organization case definition for influenzalike illness “to capture cases in which the physician may suspect this illness even if a specific diagnosis code was not present.” Their control conditions included exposure to urinary tract infections and back pain.
Doctor visits were considered unexposed if they were scheduled to start at least 90 minutes before the first influenzalike illness visit of the day. They were considered exposed if they were scheduled to start at the same time or after the first influenzalike illness visit of the day at that practice.
Researchers quantified whether exposed patients were more likely to return with a similar illness in the next 2 weeks, compared with nonexposed patients seen earlier in the day
They found that 2.7 patients per 1,000 returned within 2 weeks with an influenzalike illness.
Patients were more likely to return with influenzalike illness if their visit occurred after an influenzalike illness visit versus before, the researchers said.
The authors of the paper said their new research highlights the importance of infection control in health care settings, including outpatient offices.
Where did the exposure occur?
Diego Hijano, MD, MSc, pediatric infectious disease specialist at St. Jude’s Children’s Research Hospital, Memphis, Tenn., said he was not surprised by the findings, but noted that it’s hard to say if the exposure to influenzalike illnesses happened in the office or in the community.
“If you start to see individuals with influenza in your office it’s because [there’s influenza] in the community,” Dr. Hijano explained. “So that means that you will have more patients coming in with influenza.”
To reduce the transmission of infections, Dr. Neprash suggested that doctors’ offices follow the CDC guidelines for indoor conduct, which include masking, washing hands, and “taking appropriate infection control measures.”
So potentially masking within offices is a way to minimize transmission between whatever people are there to be seen when it’s contagious, Dr. Neprash said.
“Telehealth really took off in 2020 and it’s unclear what the state of telehealth will be going forward. [These findings] suggest that there’s a patient safety argument for continuing to enable primary care physicians to provide visits either by phone or by video,” he added.
Dr. Hijano thinks it would be helpful for doctors to separate patients with respiratory illnesses from those without respiratory illnesses.
Driver of transmissions
Dr. Neprash suggested that another driver of these transmissions could be doctors not washing their hands, which is a “notorious issue,” and Dr. Hijano agreed with that statement.
“We did know that the hands of physicians and nurses and care providers are the main driver of infections in the health care setting,” Dr. Hijano explained. “I mean, washing your hands properly between encounters is the single best way that any given health care provider can prevent the spread of infections.”
“We have a unique opportunity with COVID-19 to change how these clinics are operating now,” Dr. Hijano said. “Many clinics are actually asking patients to call ahead of time if you have symptoms of a respiratory illness that could be contagious, and those who are not are still mandating the use of mask and physical distance in the waiting areas and limiting the amount of number of patients in any given hour. So I think that those are really big practices that would kind of make an impact in respiratory illness in terms of decreasing transmission in clinics.”
The authors, who had no conflicts of interest said their hope is that their study will help inform policy for reopening outpatient care settings. Dr. Hijano, who was not involved in the study also had no conflicts.
FROM HEALTH AFFAIRS
Bronchitis the leader at putting children in the hospital
About 7% (99,000) of the 1.47 million nonmaternal, nonneonatal hospital stays in children aged 0-17 years involved a primary diagnosis of acute bronchitis in 2018, representing the leading cause of admissions in boys (154.7 stays per 100,000 population) and the second-leading diagnosis in girls (113.1 stays per 100,000), Kimberly W. McDermott, PhD, and Marc Roemer, MS, said in a statistical brief.
Depressive disorders were the most common primary diagnosis in girls, with a rate of 176.7 stays per 100,000, and the second-leading diagnosis overall, although the rate was less than half that (74.0 per 100,000) in boys. Two other respiratory conditions, asthma and pneumonia, were among the top five for both girls and boys, as was epilepsy, they reported.
The combined rate for all diagnoses was slightly higher for boys, 2,051 per 100,000, compared with 1,922 for girls, they said based on data from the National Inpatient Sample.
“Identifying the most frequent primary conditions for which patients are admitted to the hospital is important to the implementation and improvement of health care delivery, quality initiatives, and health policy,” said Dr. McDermott of IBM Watson Health and Mr. Roemer of the AHRQ.
About 7% (99,000) of the 1.47 million nonmaternal, nonneonatal hospital stays in children aged 0-17 years involved a primary diagnosis of acute bronchitis in 2018, representing the leading cause of admissions in boys (154.7 stays per 100,000 population) and the second-leading diagnosis in girls (113.1 stays per 100,000), Kimberly W. McDermott, PhD, and Marc Roemer, MS, said in a statistical brief.
Depressive disorders were the most common primary diagnosis in girls, with a rate of 176.7 stays per 100,000, and the second-leading diagnosis overall, although the rate was less than half that (74.0 per 100,000) in boys. Two other respiratory conditions, asthma and pneumonia, were among the top five for both girls and boys, as was epilepsy, they reported.
The combined rate for all diagnoses was slightly higher for boys, 2,051 per 100,000, compared with 1,922 for girls, they said based on data from the National Inpatient Sample.
“Identifying the most frequent primary conditions for which patients are admitted to the hospital is important to the implementation and improvement of health care delivery, quality initiatives, and health policy,” said Dr. McDermott of IBM Watson Health and Mr. Roemer of the AHRQ.
About 7% (99,000) of the 1.47 million nonmaternal, nonneonatal hospital stays in children aged 0-17 years involved a primary diagnosis of acute bronchitis in 2018, representing the leading cause of admissions in boys (154.7 stays per 100,000 population) and the second-leading diagnosis in girls (113.1 stays per 100,000), Kimberly W. McDermott, PhD, and Marc Roemer, MS, said in a statistical brief.
Depressive disorders were the most common primary diagnosis in girls, with a rate of 176.7 stays per 100,000, and the second-leading diagnosis overall, although the rate was less than half that (74.0 per 100,000) in boys. Two other respiratory conditions, asthma and pneumonia, were among the top five for both girls and boys, as was epilepsy, they reported.
The combined rate for all diagnoses was slightly higher for boys, 2,051 per 100,000, compared with 1,922 for girls, they said based on data from the National Inpatient Sample.
“Identifying the most frequent primary conditions for which patients are admitted to the hospital is important to the implementation and improvement of health care delivery, quality initiatives, and health policy,” said Dr. McDermott of IBM Watson Health and Mr. Roemer of the AHRQ.